You are on page 1of 61

● Agency

○ Issue: How does an agency relationship arise?


○ Method of analysis:
■ Determine which of the following three types or problems is involved.
● 1) Is the problem between the agent and the principal?
● 2) Does it involve a third party trying to hold the principal to an agreement based on
the agent’s conduct or on an express agreement?
● 3) Does it involve a third party trying to hold a principal liable for the agent’s torts?
■ Burden of Proof: On the party asserting the principal-agent relationship
■ RST 3rd of Agency, Section 1.01
● Manifestation of consent (assent) by principal
○ Note: The manifestation of consent standard is objective, it does not matter
what the principal truly intended, but rather depends on what the agent
believed the principal intended.
● That the agent shall act on principal’s behalf and subject to principal’s control.
● Consent (assent by the agent so to act for the principal) by the agent so to act for
the principal.
■ Gorton v. Doty (How does the agency relationship arise)
● Facts: Defendant offered car to coach. Car accident and son was injured. Defendant
told coach that he could use the car if he drove it.
● Rule: An agreement may result in the creation of an agency relationship although the
parties did not call it an agency and did not intend the legal consequence.
○ An agency relationship results when one person allows another to act on her
behalf and is subject to her control.
■ Direction that only the coach could drive car = sufficient to establish
control/agency relationship.
○ There does not have to be a formal written agreement. The court will look at
the course of dealings between the parties.
● Dissent: This was a gratuitous bailment/loan. No benefit to the principal. Condition
was simply a precaution.
● How could the defendant prevent liability?
○ 1) Disclaimer. (However, sometimes courts set these aside.)
○ 2) Insurance- Purchase insurance that will cover the negligence of another
driver who takes control of her car.
○ 3) Set up a leasing agreement.
■ Rst 2nd of Agency, Section 14O (Agency v. Creditor-Debtor)
● Creditor who assumes control of his debtor’s business…. May become a principal…
with liability for acts and transactions of debtor in connection with his business.
● Veto power OK
○ Intent to become an agent doesn’t matter.
■ Rst 2nd of Agency, Section 14K(Agency v. Buyer-Supplier)
● One who contracts to acquire property from a third person and convey it to another is
the agent of the other only if agreed that he is to act primarily for the benefit of the
other and not for himself.
1
○ Factors demonstrating supplier relationship rather than an agent:
■ Supplier (wholesaler) receives fixed price for property.
■ Supplier acts in own name and receives title to property later
transferred.
■ Supplier has an independent business. (Warren was only purchasing
grain for Cargill.)
■ Jenson v. Cargill (Creditor-Debtor)
● Facts: Plaintiff farmers sold grain to Warren. Defendant, Cargill, extended credit to
Warren. Warren appointed Cargill its grain agent. Cargill monitored farms activities,
controlled contract obligations and helped w/ funding.
● Rule: A creditor who assumes control of his debtor’s business may become liable as
principal for the acts of the debtor in connection w/ the business by virtue of its
course of dealing.
● Analysis: Cargill manifested assent by directing Warren to implement its
recommendations.
○ Cargill’s “paternalistic” interference w/ internal affairs established agency.
○ Cargill’ had a “first right of refusal.”
○ Creditor’s assuming control/management of debtor = agency relationship.
○ Issue: Attribution Rules- Did the agent have the authority to bind the principal?
○ Method of analysis:
■ What type of agency?
● Actual
○ Express
■ Rst 2nd §7- Power of an agent to affect legal relations of the principal
in accordance with P’s manifestation of consent to the Agent.
■ Rst 2nd §26- Creation of Authority Requires:
● A. Objective Manifestation of Principal
● B. Agent’s reasonable interpretation of that manifestation.
● C. Agent’s belief he is authorized to act for the principal.
○ Implied (See Mill Street Church v. Hogan)
■ Rst 2nd §35- When incidental authority is inferred.
● Acts which are incidental, usually accompany, or are
reasonably necessary to accomplish a transaction.
● Fills the gaps in express authority.
● Apparent (See Three-Seventy Leasing Corporation v. Ampex Corporation)
■ Restatement 2nd of Agency, §§8 & 27 – Apparent Authority;
Creation of Apparent Authority
● Objective manifestation from one party (apparent principal)
● Which reaches a third party
● Causing the third party to reasonably believe that another
party (apparent agent) is authorized to act for the apparent
principal
■ Restatement 3rd of Agency, §2.03
● 3P reasonably believes actor has authority to act on behalf of P
and that belief is traceable to P’s manifestations
2
■ Restatement 2nd of Agency, §26
● Manifestation may consist of “written or spoken words or
other conduct”
● Examples:
○ Direct communications from principal by letter/word
of mouth
○ Authorized statements of the agent
○ Documents or other indicia of authority given by the
principal to the agent
○ Communications from third persons who have heard of
the agent’s authority from authorized or permitted
channels of communication
○ Appointing a person to a position like manager or
treasurer which carries with it generally recognized
duties
○ Communication to the public through signs or
advertising
● Continuously employing the agent

● Inherent (See Watteau v. Fenwick (finding), Nogales v. ARCO)


■ Arises when there is no actual or apparent authority. This commonly
happens when there is an undisclosed principal.
■ Doctrine imposes enterprise liability on the enterprise that stands to
benefit from the agency relationship.
■ Restatement 2nd of Agency, §§8A, 161,194, 195 (deals with
undisclosed principals)
● Principal liable for acts done on his account that usually
accompany or are incidental to transactions agent authorized
to conduct.
● Exists for the protection of persons harmed by or dealing with
a servant or other agent.
■ Restatement 3rd of Agency Approach
● Declines to recognize inherent authority
● BUT states rule for undisclosed principals that achieves the
same result:
● §2.06 (2) Liability of Undisclosed Principal: “Undisclosed
principal may not rely on instructions given an agent that
qualify or reduce the agent’s authority to less than the
authority a 3P would reasonably believe the agent to have
under the same circumstances if the principal had been
disclosed.”
■ Covers actions by General Agent or General Manager
● Restatement 2nd Agency §3(a): General Agent is an agent
authorized to conduct a series of transactions involving
continuity of service. (A general agent with actual authority to
act can bind an undisclosed principal for acts forbidden/not
authorized to a third party.)
● Restatement 2nd Agency §3(b): Special agent is an agent
authorized to conduct a single transaction or a series of
3
transactions no involving continuity of service. (No inherent
authority.)
● Ratification (See Botticello v. Stefanovicz)
○ Restatement 2nd of Agency, Section 82, 83, 91
■ Retroactive approval of a previously unauthorized act
■ Affirmance through words, conduct, silence indicating consent
● Note: affirmance can be express or implied
■ Requirement of intent and knowledge of all material facts
■ Restatement 3rd of Agency, Chapter 4 – Ratification
■ Timing: Cannot follow events that would cause ratification to have
adverse/inequitable effects on 3P
○ Policy grounds can be used (Hypo- P retroactively approves a R.E.
transaction after land has been destroyed by fire. A court would likely not
allow this.)
■ Mill Street Church v. Hogan (Implied Authority)
● Facts: Mill Street Church hired Bill Hogan to paint. Bill had worked for Church and
had acquired help in the past. Church had authorized another helper but did not
require it. Bill hired brother Sam (plaintiff) Plaintiff fell and was injured.
● Holding: Implied authority to hire Sam because 1) prior dealings had allowed Bill
to hire people in the past 2) discussion about hiring someone and 3) he was hired to
paint the church and was given authority to do whatever he need to get that done.
● Analysis: Implied authority can be asserted for actions which are reasonably
necessary to accomplish task.
■ Three-Seventy Leasing Corp v. Ampex Corp (Apparent Authority)
● Facts: Salesperson, who was employed by defendant, accepted an offer by signing
the agreement. Joyce thought he was an agent. Contract feel through when
defendant’s credit failed.
● Holding: Apparent authority to enter into contract because of an objective
manifestation and reasonable belief on behalf of third party.
● Absent knowledge on the part of third parties to the contrary, an agent has the
apparent authority to do those things which are usual and proper to conduct the
business which he is employed to conduct.
● Analysis: A salesperson binds his employer to a sale if the employer agrees to the
sale in a manner that would lead one to believe that a sale has been consummated.
■ Watteau v. Fenwick (Inherent authority- found)
● Facts: Humble gave ownership of bar to Fenwick (Defendant), but continued to
manage bar. (Defendant) kept Humble’s name. Humble had limited ordering
authority but went ahead and ordered cigars from plaintiff.
● Holding: An undisclosed principal is liable for the acts of an agent taken in the
ordinary course of business even if the principal did not authorize the agent to act.
■ Nogales v. Arco (Inherent authority)
● Three situations where this power exists:
○ (1) A general agent does something similar to what he is authorized to do, but
in violation of orders;

4
○ (2) An agent acts purely for his own purposes in entering into a transaction,
which would be authorized if he were actuated by a proper motive;
○ (3) Agent is authorized to dispose of goods and departs from the authorized
method of disposal.
■ Botticello v. Stefanovicz (Ratification)
● Facts: Husband offers option to purchase on property, which he held as a tenant in
common with his wife.
● Holding: Not an automatic rule that husband can dispose of assets without wife’s
consent. No facts that indicated Mary ratified the deal or that she had knowledge of
all of the facts.
● Analysis: A statement that one will not sell for less than a certain amount is not
ratification of an agreement to sell for that amount.
● The fact that one spouse tends to business matters more than another does not
establish a principal agent relationship.
■ Estoppel
● Restatement 2nd of Agency, §8B (Hoddeson v. Koos)
○ Principal allows another (who has no authority) to create appearance of
authority and does not correct the misimpression
○ Reasonable belief by third party
○ Change in position of third party (reliance)
● Restatement 3rd of Agency, §2.05 (Hoddeson v. Koos)
○ Liable to 3P who has made a detrimental change in position due to belief it
was for another person’s account
○ Such person intentionally or carelessly caused such belief
○ With notice of such belief, such person did not take reasonable steps to notify
them of facts
● Hoddeson v. Koos Bros. (Agency by Estoppel)
○ Facts: Plaintiff was shopping at defendant’s furniture store. A fake salesman
collected money from her.
○ Holding: Where a businessman fails to exercise the appropriate level of care
to protect clients from an unrelated individual who holds themselves out as
an agent of the businessman due the businessman’s lack of duty of care, and a
client relies upon an offer from such individual, the client can assert agency
by estoppel to hold businessman/proprietor liable.
■ Liability of agent to third parties in contract
● Agency- Liability of Agent to third parties in contract
○ Partially disclosed principal- Restatement 2nd of Agency, Section 321 –
Person purporting to make a contract with a third party for a partially
disclosed principal (defined in Section 4) is a party to the contract
■ Disclosed principal (Section 4)
● Agents are relieved of liability to third parties when there is a
disclosed principal
■ Undisclosed principal (Section 4)
● Agents are liable to third parties in situations with undisclosed
principals.
■ Effect of contract
● Atlantic Salmon v. Curran (Liability of agent to third parties in contract)

5
○ Facts: Atlantic Salmon (P) sold salmon to Curran (D) acting on behalf of
Boston International Seafood Exchange. Actually, Curran (D) was acting on
behalf of Marketing Designs, Inc., but never disclosed the identity of this
principal to Atlantic Salmon (P). Atlantic Salmon (P) sued Curran (D) for
amounts owed under the Ks. (D) tried to place liability on Marketing
Designs.
○ Holding: (D) who made a contract on behalf of a partially disclosed principal
was liable as an agent.
○ Analysis: Duty to disclose agency falls upon agent falls upon agent if he
desires to avoid personal liability.
○ Not enough that the party has the means of ascertaining the mean of the
principal. The agent must
■ a) either bring to him actual knowledge; or
■ b) that to which a reasonable person is the equivalent to knowledge.
■ Liability of Principal to Third Parties in Tort
● Agency- Liability of principal to third parties in tort
○ Servant v. Independent Contractor, Restatement 2nd Agency, Section 2
(Humble, Hoover)
■ Right of principal to control “physical conduct” of servant
■ No such right of physical control of independent contractor
○ Respondeat Superior Liability, Restatement 2nd of Agency, §219
■ Master is subject to liability for torts of his servants committed in the
scope of employment
○ Definition of Servant, Restatement 2nd of Agency,§220
■ Extent of control under agreement over details, distinct business, skill,
specialist function, supplying of tools and place of work, length of
employment, method of payment, part of regular business, belief of
parties, whether principal in business
● Humble Oil & Revining Co. v. Martin (Servant v. Independent Contractor-
Master/servant relationship found)
○ Facts: Martin (P) was injured when a car rolled out of a service station
operated by Schneider and owned by Humble Oil & Refining Co. (D).
Martin (P) sued Humble (D) in negligence to recover damages for personal
injury. (Significant facts: Humble paid for operational expenses, Schneider
was required to make reports and perform duties as required by Humble,
main object of the business was the marketing of Humble’s products with
title remaining in Humble until delivery to costumer, Humble had strict
financial control, Schneider had little to no business discretion other than
general supervisor roles, Humble controlled hours of operation, Humble
could terminate commission-agency agreement at will.)
○ Holding: Master-servant relationship is established beacuse Humble
mandated/controlled much of the day-to-day operations of the station,
Humble had physical control =this is “master-servant” element
● Hoover v. Sun Oil (Servant v. Independent Contractor- Independent Contractor
relationship)
○ Facts: Hoover (P) was injured at gas station owned by Sun Oil (D) and
operated by Barone. Accident a result of service employee’s negligence.
(Significant facts: Lease could be terminated by either party, Barone could
sell competitor’s products, Barone didn’t have to make written reports to Sun,

6
Barone assumed overall risk of profit or loss, Barone didn’t have to follow
advice from Sun reps.)
○ Holding: Barone was not an agent of Sun Oil (D) because (D) did not control
the day-to-day operations of the station. Barone was an independent
contractor.
● Franchises
○ Franchise = licensing system
○ Franchisor obligations = license use of valuable name and “system”
○ Franchisee obligations = payment of royalties & advertising fees; obligations
to operate within system (embodied in operating manual and contract)
○ Legal aspects: State contract law and franchise regulation (disclosure)
○ Franchisor-franchisee relationship can give rise to liability claims
against the franchisor for franchisee negligence
● Murphy v. Holiday Inn, Inc.
○ Facts: Murphy (P) suffered personal injuries as a result of a fall at a
particular Holiday Inn, on property owned by Betsy-Len. Murphy (P) sued
the franchisor, Holiday Inns, Inc. (D), in negligence to recover damages for
personal injuries. (Significant facts: provisions did not give Holiday Inn
control over day-to-day operations or daily maintenance of premises, no
power to control business expenditures, no power to fix costumer rates, no
power to demand a share of the profits, no power to hire/fire employees,
determine employee wages/working conditions/standards.)
○ Holding: The licensing agreement/franchise contract did NOT create an
agency relationship between Holiday Inn (D) and Betsy-Len.
○ Analysis: If only getting licensing fees and not a share of the profits, argue
no agency relationship.
○ It doesn’t matter if in a contract the parties call or don’t call their relationship
a principal-agent relationship. What matters is how the actual relationship
works.
■ Tort liability and apparent agency
● Restatement 2nd of Agency, §267 (Ira Bushey, Miller v. McDonald’s)
● One who represents another as his servant or agent
○ Causing a third party to justifiably rely on care or skill of such agent
○ Is liable for harm caused by lack of care or skill of such agent
● Restatement 3rd of Agency,§7.08
○ A principal is subject to vicarious liability for a tort committed by an agent in
dealing or communicating with a third party on or purportedly on behalf of
the principal when actions taken by the agent with apparent authority
constitute the tort or enable the agent to conceal its commission.
● Miller v. McDonald’s Corp
○ Facts: Miller (P) sued McDonald’s (D) for injuries. Restaurant was owned
by 3K restaurants. (The franchise agreement between D and 3K laid out
detailed instructions on how the restaurant was to be operated. In fact, D
periodically sent inspectors to the restaurant to ensure conformity with D’s
standards.
○ (A jury could find that D retained sufficient control over 3K’s daily
operations such that an actual agency relationship exited.)
○ Holding: An actual agency relationship between (D) McDonald’s and 3K
existed that could hold (D) liable for injuries. However, even if the actual
7
agency didn’t hold up, there was apparent agency because a jury could find
that McDonald’s held 3K out as their agent in a way that the plaintiff relied
on this holding out.
■ Everything about the appearance of the restaurant identified it with D
and the common image for all of D’s restaurants. D has created this
image with national advertising, common signage, menus, and
uniforms, etc. P has testified that she went to the restaurant under the
belief that it would have the same quality of service and care as other
McD’s.
■ Agency Scope of Employment
● Restatement 2nd of Agency, §228(1) – When conduct is within the scope of
employment (Ira Bushey)
○ Of the kind he is employed to perform
○ Within authorized time and space limits
○ Purpose to serve the master
○ If force intentionally used, use of force not unexpectable by the master
● R2nd of Agency §229 factors for when conduct is within the scope of employment
○ Act commonly done by servants
○ Time, place & purpose of act
○ Previous dealings between P & A
○ How business is apportioned between different servants
○ Act outside of enterprise of master or not entrusted to servant
○ Master expectation that act will be done
○ Similarity to act authorized
○ Instrumentality furnished by master
○ Departure from normal methods
○ Act is seriously criminal
● Ira Bushey & Sons v. United States
○ Facts: Drunk sailor turned valves on way back from bar—damaged drydock.
○ Holding: Court found it was not so unforeseeable as to make it unfair to
charge the government for cost of the action.
○ The employer should be held to expect risks to itself and to the public that
would arise out of the behavior of its employees. It was foreseeable that
crewmen crossing the drydock to return to the ship would cause damage.
The actions were not entirely caused by Δs personal life.However, there is a
limit. Had Δ shot a guard or burned down a bar, US would not be liable
(actions must be related to work life, not personal life).
■ Agency- Liability for Torts of Independent Contractor
● Servant vs. Independent Contractor
○ R2d Agency, Sections 2, 220
○ General rule: One who hires an independent contractor is not liable for such
contractor’s negligent actions
○ Exceptions:
■ 1) retain control of manner and means of doing work;
■ 2) engages an incompetent contractor;
■ 3) inherently dangerous activity
● Restatement 3rd of Agency, §7.07

8
○ Employer vicariously liable for torts of employee acting within scope of
employment
○ Scope of employment = performing work assigned by employer or engaging
in course of conduct subject to employer’s control
○ Scope of employment ≠ occurring within independent course of conduct not
intended by employee to serve any purpose of the employer
● R2d Torts Section 416: One who hires an independent contractor to conduct
inherently dangerous activity requiring precautions is liable if contractor is negligent
in taking precautions
○ § 520, Ultra-hazardous activities: Strict liability when an activity is ultra
hazardous. Ultra-hazardous activity is one which (a) necessarily involves a
serious risk of harm to the person, land, or chattels of others which cannot be
eliminated by the exercise of the utmost care, and (b) is not a matter of
common usage.
● Majestic Realty v. Toti Contracting
○ Facts: Majestic owned property which Bohen rented. Authority acquired
property adjacent to current building to establish a public parking area, which
required demolition of several buildings. Contract entered into and Toti
agreed to do the work. In doing so, section fell on Majestic’s roof. Evidence
showed that Toti was using improper demolition methods.
○ Holding: Defendant is liable for negligent acts of independent contractor.
○ Analysis: · BASICALLY, although a master/employer is NOT liable for
torts of independent contractor, he/she will be held liable when the contractor
is performing inherently dangerous work and precautions measures are not
taken.
■ Courts have equated nuisance per se with activities that are inherently
dangerous. The Rest. Of Torts considers an activity inherently
dangerous if it can only be carried out safely by the exercise of special
skill and care, and it involves grave risk of danger to persons or
property if negligently done.
■ Distinction between nuisance per se and ultra hazardous activities
which impose strict liability —necessarily involves a series risk of
harm to the person, land or chattels of others which cannot be
eliminated by the exercise of utmost care and is not a matter of
common usage
○ Agency- Fiduciary Relationship
■ Restatement 2nd Agency §1 – “Agency is … [a] fiduciary relationship”
● A fiduciary relationship is one involving trust and confidence
● Agent must place principal’s interests over her own
■ Reading v. Regem
● Facts: (P) Reading used his military uniform as a servant of the crown to obtain
extra money.
● Holding: It makes no difference that in this case there was no fiduciary relationship
and P was not acting within course of his employment. Nor does it matter that there
was no loss to the Crown. Rule against secret profits - If a servant unjustly enriches
himself by virtue of his service without his master’s sanction, the law says that he
shall not be allowed to keep the money, but it shall be taken from him and given to
his master.
9
● Moonlighting exception: If you’re doing something off hours in which you profit
and it isn’t a breach of fiduciary duty, then the principal has no claim to the money.
Essentially, you cannot use the assets of your principal to make a profit (ie- a
uniform). The principal may be entitled to contract damages if there is a breach of
contract but not to the money obtained through moonlighting. (U.S. retired general
getting royalties from memeoir.)
○ Agency- Duty of Loyalty
■ Restatement 2nd of Agency, §387 – Duty of Loyalty
● A is subject to a duty to act solely for the benefit of the P in all matters relating to the
agency
■ Restatement 2nd of Agency, §388 – Duty to Account for Profits Arising out of Employment
● If A makes a profit in connection with transactions conducted by him on behalf of
the P, A must turn over profit to P.
■ Restatement 2nd of Agency, §404 – Liability for Use of Principal’s Assets
● A must pay over profit if uses assets of P in violation of a duty
● A not liable for profits made by use of time to be devoted to principal unless he
violates duty not to act adversely or in competition with P
■ Restatement 3rd of Agency, §8.02 – Material Benefit Arising out of Position
● Agent has a duty not to acquire a material benefit from a third party in connection
with transactions conducted or other actions taken on behalf of the principal or
otherwise through the agent’s use of his position
■ Restatement 3rd of Agency, §8.05 (1) – Use of Principal’s Property
● An agent has a duty not to use the property of the principal for the agent’s own
purposes or those of a third party
○ Agency-Fiduciary Duty- No Competition
■ Restatement 2nd of Agency, §393 – Competition as to Subject Matter of Agency
● Agent under a duty not to compete with principal concerning the subject matter of
the agency
■ After termination of agency, barrier to competition ends §396(a)
● Exceptions: cannot use confidential information, no deceit, non-compete contract
clauses
■ Compare R3d of Agency §8.04: during the agency, A under a duty to refrain from
competing with P and from taking action to assist P’s competitors; A may take action to
prepare for competition following termination of agency
■ General Automotive v. Singer
● Facts: Plaintiff hired defendant. Defendant helped plaintiff’s business grow but
defendant reasoned that plaintiff’s shop was unable to fill some orders that required
different machinery/capacity. Defendant did not notify plaintiff of these orders and
instead filled these orders himself through other machine shops, keeping all the
profits from the sideline business.
● Holding:
○ D had a fiduciary duty as agent of P to exercise the utmost good faith and
loyalty. Instead, he not only acted in his own self-interest, but he acted
adversely to the interests of P.
○ D had a duty to disclose the existence of the other orders to P. it was then in
P’s discretion to refuse to accept the orders or to fill them, if possible, or to
sub-job them to another ship. The profit, if any, would belong to P. if P knew
of these other orders, P may have decided to expand its operations, install

10
suitable equipment to complete the orders, or make other arrangements that
would allow P to reap the profits.
○ By failing to disclose the existence of these secret orders, D violated his
fiduciary duty to act solely for the benefit of P, and therefore he is liable to P
for the amount of the profits he earned.
○ Agency-Fiduciary Duty-Confidential Information
■ Restatement 2nd of Agency, §395: During agency, A has duty not to use or disclose
confidential info
■ Restatement 2nd of Agency, §396(b) After termination of agency, A has duty not to use…in
competition with the P or to his injury…trade secrets, written lists of names, or other similar
confidential matters…
■ Compare R3rd of Agency, §8.05(b)
■ Town & County v. Newbery
● Facts: P’s enterprise is a house and home cleaning by contract w/ individual
householders. While employed by P, Ds conspired to terminate their employment,
form a business of their own in competition w/ the P and solicit Ps customers for
their business. There was a customer list.
● Holding: Defendants owe Plaintiff the profits they made from customers taken from
Plaintiff, but they do not have to cease their operations. Defendants had a duty to
protect Plaintiff’s trade secrets and are prohibited from profiting from the secrets
even after their employment ended. The customer listing was formulated through
much effort on behalf of Plaintiffs, but their methods of cleaning a house were
nothing so secretive as to justify prohibiting Defendant’s from continuing their
cleaning services.
● Partnership:
○ UPA, Section 6(1): An association of two or more persons to carry on as co-owners a business for
profit
○ Uniform Partnership Act: Has been adopted by most states, so that the provisions governing
partnerships are usually a part of state statutory law, rather than common law.
○ Missouri still uses the UPA from 1914.
○ Existence of a partnership
○ Sharing of Profits, Equal Control Rights
● UPA, Section 7(4)
○ Share of profits is prima facie evidence of partnership but not if inference
shall be drawn if such profits were received as a payment:
■ …as wages of an employee…or as interest on a loan… (see other
exceptions)... as the consideration for the sale of a good-will of a
business.
● UPA, Section 18(e)
○ All partners have equal rights in the management and conduct of the
partnership business
● UPA, Section 18(a)
○ Partners share equally in profits and losses
○ Partners compared with employees
■ Fenwick v. Unemployment Compensation Commission
■ Facts: Plaintiff was an at-will cashier/receptionist for defendant’s beauty shop. Written
agreement was drafted by attorney and called a partnership.
● 1. Did the parties intend to become partners? (No.)
11
● 2. Was there a right to share in profits? (Yes)
● 3. Obligation to share in losses (No.)
● 4. Ownership and control of the partnership property and business (No)
● 5. Power in administration—control in management (No).
● 6. Language of Agreement (No).
● 7. Conduct of parties to agreement towards third parties indicating partnership (No).
■ Holding: Although called a partnership, not binding b/c not enough factors were
sufficiently satisfied and income was received as wages, not partnership share. The court
held that even though there was a profit sharing plan in place, this was not a partnership and
was instead an agreement for an alternative compensation method.
■ Judicial factors used in Fenwick
● Intention
● Sharing of Profits
● Sharing of Losses
● Contribution of Capital and Share in Capital Upon Dissolution
● Control of Business
● Language in Agreement
● Conduct towards Third Parties
○ Partners compared with lenders
■ UPA, Section 15
● All partners are jointly and severally liable for debts and obligations of the
partnership
■ Martin v. Peyton
■ Facts: Plaintiff’s claim that defendants became partners in an investment firm when
they invested large amount of money in it.
■ Holding: Court held that although defendants were kept advised on matters and
allowed to inspect the books, these were more the actions of an investor and not
partner. No ability to bind operation in contract—explicitly not called a
partnership.
■ Analysis: However, just because there isn’t an explicit partnership agreement
doesn’t mean that a partnership does not exist.
■ Sharing of profits is not conclusive evidence of a partnership.
○ Partnership by Estoppel
■ UPA, Section 16
● If a person represents himself as a partner in an enterprise (or allows another to so
represent him) and
● 3P relies on that representation and enters into a transaction with the supposed
partnership (“has given credit”)
● That person is liable to 3P on that transaction
■ Young v. Jones
● Facts: P’s are investors from Texas who deposited over a half million dollars in a S.
Carolina bank and the funds have disappeared. PW-Bahamas-a Bahamian affiliate of
Price Waterhouse Coopers U.S. issued an unqualified audit letter regarding the
financial statement of Swiss American Fidelity and Insurance Guaranty. PW-
Bahamas wrote the letter, and P went after PWUS b/c PWB did not have enough

12
assets. B/C letter didn’t establish actual partnership, P’s tried using partnership by
estoppel. Plaintiff tried to claim that all of the information/brochure (showing PW-
cooper as an entity with partners all over the world) suggested that PWC was a single
entity.
● Holding: Two separately organize firms, a U.S. firm and its foreign affiliate, do
NOT operate as partners by estoppel when the foreign affiliate used the firm name
and trademark. This was just a licensing agreement. Since plaintiffs couldn’t show
reliance on
brochure, no partnership by estoppel.
● Rule: If someone relies on partnership, courts may hold partners responsible.
● Analysis: Plaintiff did not rely upon apparent partnership. The element of estoppel,
giving credit, didn’t happen in this case. Credit had to be extended from PW-US, but
it wasn’t.
○ Limited Partnerships
■ California Civil Code §2483
● “A limited partner shall not become liable as a general partner, unless in addition to
the exercise of his rights and powers as a limited partner, he takes control of the
business.”
■ RULPA (Revised Uniform Limited Partnership Act) Section 303(a): Limited partner is
NOT LIABLE FOR OBLIGATIONS of limited partnership unless
● Limited partner is also a general partner OR
● Limited partner takes part in the control of the business
○ In this case, limited partner is liable to 3P who transact business with limited
partnership and who reasonably believe based on limited partner’s conduct,
that she is a general partner
■ RULPA Section 303(b): Limited partner does not participate in control solely by
consulting/advising with general partner on partnership business
■ Holzman v. De Escamilla
■ Facts: Hacienda Farms was organized as a limited partnership with Escamilla as the general
partner and Russell and Andrews as limited partners. Russell and Andrews had absolute
power to withdraw all the partnership funds in the banks without the knowledge or consent
of the general partner. Either Russell or Andrews could take control of the business from
Escamilla by refusing to sign checks for bills contracted by him and thus limit his activities
in the management of the business. The required him to resign ans manager and selected his
successor. They were active in dictating the crops to be planted, some of them against the
wish Escamilla.
■ Holding: Defendants, Russell and Andrews, shows that they took part in the control of the
business of the partnership and thus became liable as general partners.
■ Analysis:
● There are separate laws in every state for limited partnership acts. If you form a
limited partnership, you have to follow the procedures in your state.
● DIFFERENCES FROM GENERAL PARTNERSHIP
○ Formalities: Need to file certificate of limited partnership
○ Two categories of partners: GP & LP
○ Limited liability for LPs; Unlimited for GP
○ Management in GP; LPs are passive investors

13
○ Profit and Loss Sharing: LPs share in profits and losses based on their
contributions
○ Dissolution: Dissociation of LP does not dissolve
○ Fiduciary obligation of partners
■ Fiduciary Duty of Loyalty
● UPA, Section 21
○ Partner must account/hold as trustee (disgorgement) for profits/benefits
derived from any transaction connected with partnership or use of its property
● RUPA, Section 404b – Duty of Loyalty
○ Duty of loyalty to account/hold as trustee profits/benefits derived from a use
of partnership property including partnership opportunity
○ Refrain from conflict of interest transactions
○ Refrain from competing before dissolution of the partnership
● Meinhard v. Salmon
○ Facts: Meinhard (P) filed suit against Salmon (D), his business partner in a
joint venture. P was to provide investment capital, Salmon would manage the
business and the two of them would divide up the profits. After two year
lease of property, landlord approached D with an opportunity to lease a much
larger swath of property, including the current building for a much bigger
redevelopment project. D agreed and entered into the agreement without P. P
sued D for “usurping” an opportunity that should have been presented to the
joint venture.
○ Holding: The new lease came within D’s fiduciary obligation to his joint
venture partner as a joint venture opportunity
■ a. Salmon, as the managing partner, owed Meinhard, as the
investing partner, a fiduciary duty and this included a duty to inform
Meinhard of the new leasing opportunity. Joint venturers owe each
other the highest duty of loyalty – "Not honesty alone, but the
punctilio of an honor the most sensitive."
■ b. This decision extended the duties of partnership far beyond duties
under a contract. It determined that in such a relationship, loyalty
must be undivided and unselfish, and that a breach of fiduciary duty
can occur by something less than fraud or intentional bad faith.
○ Analysis: Duty of loyalty, at the least, requires disclosure to the joint
venture/partnership of an opportunity that is very similar to current situation.
The fiduciary duty includes the responsibility of not acting unilaterally, but
putting the interest of the business first.
■ Partnership- Fiduciary Duty of Care
● RUPA, Section 404c – Duty of Care
○ Partner must not act in a manner that is grossly negligent or reckless or
engage in intentional misconduct or knowing violation of the law
● UPA, Section 20
○ Partners shall provide on demand true and full information of all things
affecting the partnership to any partner
● UPA, Section 19
○ Partners may inspect and copy partnership’s books
● RUPA, Section 403
○ Partners may inspect and copy books and records

14
○ Partner entitled to information from other partners and partnership that is
needed for exercise of partner’s rights and duties without making demand
■ Example: Rachel and Sam are partners and Rachel is considering
selling her transferable interest to Sam. Sam learns of some
information suggesting the partnership is entering a boom period.
Rachel is unaware of that information. He must disclose that
information to Rachel even though she has not made demand.
● Partner entitled to other information upon demand
● Meehan v. Shaughnessy
○ Facts: M (P) and B (P) commenced an action to determine their rights and
liabilities after terminating their relationship with P/C (D), their former law
firm, to start a firm of their own. M and B sue PC to recover accounts
claimed owed to them under the partnership agreement. PC counterclaimed
that M and B had breached their fiduciary duties, breached the partnership
agreement, and tortuously interfered by withdrawing agents and clients from
PC
○ Holding: But there has been a breach of duty by Ds in the manner in which
they acted to take clients from the firm. To gain the consent of the firm
clients to remove them from the firm, they acted in secret and they obtained
an unfair advantage over the firm in communicating with these clients by
denying that they were leaving the firm, by preparing notices to go out
immediately to the clients, by delaying giving information to the partners’
separation committee. Further, the letter to the clients did not indicate that the
clients had a choice to remain; it simply indicated that they were leaving and
wanted permission to remove the clients’ files from the firm. Secrecy issues:
Duty owed that was breachedà disclosure of information. They were directly
asked if they were going to leave and they denied it. This is problematic.
○ Analysis: BASICALLY, in the course of terminating/dissolving a
partnership, partners still owe fiduciary duties. Major issue was with
disclosure.
● Expulsion of a Partner
○ UPA, Section 31(d)
■ Dissolution is caused without violation of the partnership agreement
by expulsion of any partner from the business bona fide in accordance
with such a power conferred by the agreement between the partners.
○ Lawlis v. Kightlinger & Gray
○ Facts: Although Lawlis (P) successfully battled his problem with alcohol
abuse, the law firm of K&G (D) voted to expel him from his senior
partnership position. Lawlis sued to recover damages for breach of K.
■ Guillotine clause: There is a clause in the partnership agreement,
which is particular to this law firm, “By 2/3 majority vote, Senior
Partners may expel any partner at any time upon such terms and
conditions as set by said Senior partners.”

15
○ Holding: When a partner is involuntarily expelled from a business, his
expulsion must be bona fide or in good faith. Partners cannot act with a
predatory purpose when expelling a partner.
○ Executive committee can expel plaintiff w/o cause because of the guillotine
clause in the partnership agreement.
○ Partnership- Financial Investment and Return
■ Partners contribute capital and/or labor
■ Financial return (UPA §18(a))
● Right to repayment of contribution
● Right to share equally in profits and surplus after payment of liabilities
● Obligation to contribute to losses sustained by partnership according to share in
profits
■ Right to indemnity against expenses and liabilities incurred in partnership business (UPA
§18(b))
○ Partnership-Property Rights
■ UPA §24 EXTENT OF PROPERTY RIGHTS OF A PARTNER
● Rights in specific partnership property
○ Partner is co-owner with partners of specific partnership and has right to use
specific partnership property for partnership purposes (but not for other
purposes)(UPA §25 (1))
● Interest in the partnership
○ Partner’s interest in partnership is share of profits and surplus and is personal
property (UPA §26)
● Right to participate in management
■ UPA § 27 ASSIGNMENT OF PARTNER’S INTEREST
● Assignee may only receive profits of assignor but may not participate in
management or require information or account of partnership transactions or look at
books unless there is an agreement with the other partners
○ Partners may only assign their economic interests in the partnership
○ Consistent with UPA §18(g): No person may become member of partnership
without the consent of all the partners.
■ Putnam v. Shoaf
● Facts: Carolyn Putnam had conveyed her partnership interest, and when she found
out that she had that she had missed out on some money during the course of the
partnership.
● Holding: She had conveyed her entire ownership interest in the partnership.
● Analysis: Once you give up your interest in the partnership, you’re done. When a
partner conveys interest in a partnership, she does not convey property held by the
partnership but only her interest in the partnership.
● The only thing you can convey is your interest.
○ The Rights of Partners in Management
■ Partnership- Default Voting Rules
● Disagreements among partners are decided by a partnership vote
● One partner = one vote, even if contributions are not equal (UPA §18e)
● Some matters are decided by majority vote = ordinary business decisions (UPA
§18h) (National Biscuit Company)
● Other matters require unanimous consent (UPA §9(3), §18g, §18h)
16
○ Assign partnership property in trust to creditors/secure payment of debt (9(3))
○ Dispose of good will of partnership (9(3))
○ Do an act making it impossible to carry on partnership’s ordinary business
(9(3))
○ Confess a judgment against partnership (9(3))
○ Submit a claim involving the partnership to arbitration (9(3))
○ Admit new partners (18g)
○ Contravene any agreement of the partners (18h)
■ This may include extraordinary matters that substantially change past
practice e.g. entering new lines of business.
● National Biscuit Company v. Stroud (18(h))
○ Facts: Stroud (D) and Freeman entered a partnership to sell groceries under
the name Stroud’s Food Center. There were no restrictions in the partnership
agreement on the management functions or authority of either partner.
Several months prior to February 1956, D notified National Biscuit Co (P)
that he would not be responsible for any additional bread delivered to the
Food Center. Nevertheless, on Freeman’s order, P delivered $171 worth of
bread over a two-week period in February. At the end of this time, D and
Freeman dissolved the partnership, and D was responsible for winding up its
affairs. D refused to pay P’s bill.
○ Holding: The acts of a partner within the scope of the partnership business
bind all partners
○ A majority of partners can make a decision and inform creditors and will
thereafter not be bound by acts of minority partners in contravention of the
majority decision. But here there could be no majority decision, as they are
equal co-partners. Hence, the partnership is liable for the debt to P.
○ Note: Had D dissolved the partnership and given P notice prior to the order
by Freeman, D would not have been personally liable for the partnership debt
to P. P would have been liable in his individual capacity.
○ Take away point: Stroud could not restrict the power and authority of
Freeman to buy bread for the partnership as a going concern, for such a
purchase was an “ordinary matter connected with the partnership
business,”(UPA 18(h)) for the purpose of its business and within its scope
because in the very nature of things Stroud was not, and could not be, a
majority of the partners.... Purchase of bread from plaintiff as a going
concern bound the partnership and his co-partner Stroud.
● Day v. Sidley & Austin
○ Facts: Plaintiff was a senior partner with a long, distinguished career.
Plaintiff worked for Defendant out of Washington, D.C. Defendant’s
executive committee inquired into merging with another firm, but partners
not on the committee were not aware of the deliberations. However, once the
firm decided to merge, the proposed merger was brought to a vote for the
partners at Defendant firm. Plaintiff voted in favor of the merger but was not
aware that the firm intended on merging the Washington offices wherein he
would share the chairmanship title with the former chair of the other firm.
Plaintiff argued that he had a contractual right to the title of chair of the
Washington office, and that the title change was intentionally kept from him

17
when he voted in favor of the merger. If he would have known of the change,
Plaintiff would not have voted in favor of the merger – and therefore there
would not have been a unanimous vote to merge – and no merger would have
taken place. Plaintiff argued that he was professionally humiliated by the title
change.
○ Holding: Defendant did not violate a fiduciary duty to Plaintiff by their
merger and subsequent title change for Plaintiff. The partnership agreement
that Plaintiff signed authorized the executive committee to appoint members
and chairpersons, so Plaintiff was aware of the possibility of a co-chair. Also,
Defendants decisions were not made to personally profit at the expense of the
firm, and their fiduciary duty does not extend to what Plaintiff proposes
■ Members of a partnership do not violate a fiduciary duty to one
partner simply because that partner believes that their position was
de-valued in any manner. If the other partners are following the
agreement provisions and they are exercising their rights in a manner
that is beneficial for the partnership over their personal interests, then
they are meeting their duty.
■ Partnership- Partners as Agents
● Each partner is an agent for partnership and binds the partnership when apparently
carrying on in the usual way the business of the partnership (UPA §9(1))
○ Exception: Partner has no authority to act for partnership in the matter and
3P knows that
● Partners are jointly and severally liable for debts and obligations of partnership
(UPA §15)
■ Partnership-Changing Management Rights by Contract
● Default rules are often changed in the following areas:
○ Delegating decisionmaking to a managing partner or executive committee
○ Weighting partnership voting to reflect pro rata contributions to capital
○ Changing requirement of unanimous consent
○ Requiring supermajority voting for important decisions
○ Right to expel partners
○ Partnership Dissolution
■ Dissolution versus Winding Up: Partnership is not terminated upon dissolution but
continues until winding up of business is completed. (UPA, Section 30)
■ Causes of Dissolution (UPA Section 31)
● Without violation of partnership agreement:
● At the end of a fixed term or with consent of all partners if partnership for a term
● By express will of any partner if partnership at will
● Upon expulsion of a partner under a clause in the partnership agreement
■ Causes of Dissolution (UPA Section 31)
● With violation of the partnership agreement, if dissolution not permitted by any other
section, by express will of any partner at any time
● Business becomes unlawful
● Death or bankruptcy of partner or bankruptcy of partnership
● Court decree under Section 32
■ Dissolution by Court Decree (UPA Section 32)
● Upon application to the court:
18
○ If partner is insane or unable to meet requirements of partnership agreement
○ If partner guilty of such conduct as prejudices carrying on the business
○ Partner willfully or persistently breaches the agreement or makes it not
reasonably practicably to carry on business with him
○ Business can only be carried on at a loss
■ Owen v. Cohen (Judicial decree of dissolution)
● Facts: On Jan 2, 1940, P and D made an oral agreement to become partners. The
agreement did not contain a statement of duration. P advanced $6,986 to the
partnership for the purpose of securing necessary equipment with the understanding
that he was making a loan that would be repaid out of the prospective profits. P and
D opened the bowling alley and it operated at a profit during the three and a half
months between its opening and the commencement of the case at bar. During this
period, the partners paid off part of the debt and each took a salary of $50 per week.
The partners also, however, began to have disputes regarding management and their
respective rights and duties under the agreement, which had a negative effect on the
profits of the business. On June 28, 1940, with much of the partnership’s
indebtedness unpaid, P sued for the dissolution of the partnership and the sale of its
assets in connection with the settlement of its affairs. P also wanted to be repaid as a
regular creditor for the start-up loan he gave to the partnership.
● Holding: Court found dissolution by judicial decree was proper because Cohen was
prejudicing (adversely affecting) the carrying on of the business. Court could also
decree that P’s loan be paid fro the proceeds of the sale of the partnership’s assets.
■ Duration of partnership
● At will - no limitation on duration; default rule
● Express term – “Together for [5, 10, 15…]years”
● Implied term
○ Until certain sum of money earned
○ One or more partners recoup investment
○ Certain debts are paid
○ Certain property disposed of on favorable terms
■ Page v. Page
● Facts: P (HB)and D (George) are brothers partners in a linen supply business.
Partners entered into an oral agreement/ W/in first 2 yrs both partners contributed
approx 43 K to begin the business. The business was unprofitable. Partnerships
major creditor was a corporation, wholly owned by P that supplies the linen and
machinery necessary for day-to day operation of business. Corp holds a 47 K
demand note of the partnership. Partnership operations began to improve. Despite
improvement, P wants to terminate the partnership.
● Issue: Whether P (HB) has the power to dissolve the partnership
○ (i) P wants to run business on his own
○ (ii) Disagreement whether this is a term partnership or an at will
partnership, D thinks there is an implied term.
● Holding: In this case, P has the power to dissolve the partnership by express notice
to D. No showing in the record of bad faith or that the improved profit situation is
more than temporary. Remanded to make this determination.
● Rationale: UPA-partnership can be dissolved by express will of partner but must be
done in good faith.

19
○ (i) A partner at will is not bound to remain in a partnership, regardless of
whether the business is profitable or unprofitable.
○ (ii) A partner may not however, by use of adverse pressure “freeze out” a co-
partner and appropriate the business to his own use.
○ (iii) A partner may not dissolve a partnership to gain the benefits of the
business for himself, unless he compensates his co-partner for his share of the
prospective business opportunity.
● 2 kinds of evidence people introduce when arguing for an implied term
○ a. Past agreements: Entered into a partnership in the past, and in that prior
partnership we agreed that there would be a term
■ i. Ct says this doesn’t work in this case b/c past partnership was not
same kind of partnership, it was a limited partnership
○ b. Outstanding debt/loan- agreed we would stay together until we paid that
off
■ i. Cts says that you only agreed that you hope the partnership would
be profitable; you did not agree that you would stay together through
the losses. Therfore, not entitled to protection you might receive
under UPA section 38
● How would fiduciary duty law protect George?
○ His right to give notice for any reason is limited by his obligation under fid
duty law to compensate George for his share of the opportunity if George can
show there was a breath of fid duty. So, even though HB has the right to
terminate the partnership under UPA law w/ notice, he still would have the
responsibility to give the other partner the full value
■ Liquidation or Continuation
● Right to Require Liquidation, UPA Section 38(1)
○ If dissolution caused in any way except in breach of agreement, partner may
request liquidation.
■ Default rule is that upon dissolution caused in any way (except in
breach of partnership agreement) any partner may request liquidation
■ In practice, partners often agree to continue the business because
liquidation will not produce maximum value to partners
● Right to Damages and to Continue the Business UPA Section 38(2)
○ If dissolution in violation of partnership agreement occurs: non-breaching
partner may claim for damages against breaching partners and may continue
the business and possess the partnership property for that purpose
○ If business continued, breaching partner entitled to receive value of her
interest less damages but not including good will
■ Pav-Saver Corp v. Vasso Corporation
● Facts: Pac-Saver owned patent and entered into partnership with Vasso Corp
for manufacture and sale of machines. Pac-Saver wanted to terminate
partnership and get patents back. However, partnership agreement said it is a
permanent parntnership.
● Holding: Patent stays with Vasso and Mr. Dale has to pay liquidation
damages because UPA says, “the value of the goodwill of the business shall
20
not be considered.” Pac Saver’s good will evidence of the value of patents in
valuing its interest in partnership agreement properly rejected. Here, the UPA
trumped the contractual agreement. The only value that P could show was
attributable to patent/trademark rights, which fell under the heading of good
will, which is excluded from the UPA.
● Dissent: (Wagner agrees) UPA should not trump K decisions.
■ Distribution following Dissolution
● Rules for Distribution – Payment of Liabilities, UPA Section 40(b)
○ Payment to creditors other than partners
○ Payment to partners other than for capital or profits
○ Payment to partners for capital
○ Payment to partners for profits
● Rules for Distribution – UPA, Section 40(d)
○ Partners must contribute the amount necessary to satisfy the liabilities in
Section 40(b)
○ As provided in Section 18(a)
● Capital Accounts
○ Partnerships establish capital accounts for each partner where the following
are recorded:
■ Additions: initial capital contributions and additional capital
contributions, fair market value of contributed asset at time of
contribution, profits allocated to partners from ongoing activities
■ Subtractions: interim withdrawals of capital, losses allocated to
partners from ongoing activities
■ Post-contribution appreciation or depreciation of contributed asset
does not affect capital accounts.
○ In a rightful dissolution, where the partnership is liquidated, the assets are
sold:
■ Out of the proceeds, partners receive value of their capital accounts
after creditors and partner loans are paid off.
■ Profits are what remains and that is divided according to the default
rule of equal sharing or as agreed by the partners.
○ In a wrongful dissolution, settling among partners is the same except
breaching partner share is decreased by damages under UPA §38 (2)(a)(II).
● Kovacik v. Reed - (Minority viewpoint)
○ Facts: . Kovick told Reed that he had a chance to remodel some kitchens and
asked Reed to become his job superintendent and estimator. Kovick invested
about $10,000.00, and told Reed that if he agreed to be superintendent, he
would share profits 50-50. No contemplation of losses, Reed worked on jobs,
but contributed no funds. Kovick provided the financing. When Kovick told
Reed that the venture had lost money, Kovick demanded that Reed contribute
to the amounts that he had advanced beyond the income he had received.
Reed claimed that he never agreed to be liable for losses and refused to pay.
○ Holding: A party who has contributed only his services and not capital to a
joint venture is not liable for a portion of the ventures losses.
○ Note: Minority viewpoint-most other cts would have awarded Mr. K is equal
share in terms of the losses, the RUPA § 401, explicitly addresses this
21
situation and the losses that exist and how they should be dealt with- rejects
the rule of Kovacick vs. Reed
■ Buyout Agreement
● A buy-out, or buy-sell, agreement is an agreement that allows a partner to terminate
his/her relationship with the other partner AND receive a cash payment (or series of
payments or some assets of the firm) in exchange for his/her interest in the firm
○ May be entered into upon any partner’s death, disability, or will
● Common terms in buy-out or buy-sell agreement
○ Trigger events
○ Obligation versus option to buy
○ Price
○ Method of Payment
○ Protection against partnership debts
○ Procedure for offering to buy or sell
● G & S Investments v. Belman
○ Facts: Limited partnership formed for ownership of 62 unit apartment
complex – G&S, Nordale, Jones and Chapin. Nordale began using drugs . He
insisted on turning the apts into condos and raising rent. G&S filed a
complaint asking for a judicial decree of dissolution that would allow them to
carry on business. Nordale dies, and GS files an amended complaint
requesting buy out under the terms of the partnership agreement. Belman,
representative Nordale’s estate argues that filing of the complaint acted as a
wrongful dissolution of the partnership-requiring a liquidation of the assets
and distribution of the proceeds to the partners.
○ Holding: Ct says the filing the complaint did not cause a wrongful
dissolution, but that Nordale’s conduct allowed for dissolution under UPA §
38(1). Ct based its decision on UPA § 32 (b-d) giving them the authority to
dissolve partnership and allow GS to continue partnership under § 38, buy
out provision. Under UPA § 38, you look to the partnership agreement
provision dealing w/ buyouts to determine compensation. In this case, the
partnership agreement says a partner may be bought out by a sum of this
capital account + the average profits and gains actually paid to the general
partner. Thus, the D is stuck w/ the contract provisions.
○ Essentially, because there was a contract, the partners’ right and liability are
subject to the agreements made among them.
■ Law Partnership Dissolution
● Rule against extra compensation – Partner not entitled to remuneration for acting in
the partnership business except for surviving partner receiving reasonable
compensation for his services in winding up the partnership, UPA Section 18(f)
● Jewel v. Boxer
○ Facts: On Dec 2, 1977, the law firm of Jewel, Boxer and Elkind was
dissolved by agreement of its four partners, Jewel and Leary (Ps) and Boxer
and Elkind (Ds). The partners had no written partnership agreement and
lacked any agreement as to how fees from actives cases should be allocated

22
upon dissolution. The firm partners went on to form two new firms, Jewel
and Leary, and Boxer and Elkind. Boxer and Elkind hired three associates
who had been employed by the old firm. When the old firm was dissolved, it
had several active cases. Boxer, Elkind and the three associates had been
responsible primarily for personal injury and workers’ compensation, while
Jewel and Leary had handled most of the rest. Clients executed and returned
firms and the new firms represented the clients under fee agreements into
which the clients had entered with the old firm.
○ Holding:
○ The UPA provides that a dissolved partnership continues until unfinished
partnership business is wound up. Absent an agreement to the contrary,
income generated through an old partnership’s unfinished business is
allocated among the partners according to their respective interests in the
former partnership. Unless a partner is the surviving partner, the UPA
expressly prohibits the receipt of (rule against) extra compensation,
meaning no partner can receive more than the value of his partnership interest
regardless of the level of his participation in winding up the unfinished
business.
○ Two basic fiduciary duties
■ 1. Each former partner has a duty to wind up and complete the
unfinished business of the dissolved partnership.
■ 2. No former partner may take any action with respect to unfinished
business which leads to purely personal gain
● Meehan v. Shaughnessy II
○ Facts: Upon leaving the partnership, Meehan and Boyle (Ps) violated
fiduciary duties to remaining partners. Meehan and Boyle (Ps) still
maintained that they were owed amounts under the partnership agreement for
fees
○ Holding: Rights provided by a partnership agreement, even though different
from those provided in the UPA, control the method of dividing assets upon
dissolution, provided the dissolution is not premature
■ a. The court held because the Code of Professional Responsibility
provides that lawyers cannot make an agreement that binds the right
of a lawyer to practice law after termination of the relationship (to
protect clients in retaining who they wish to have do their work), the
firm could not prevent a departing partner from removing any client
who wished to retain his services.
■ b. The damages allowed are only those that are caused by Ds’ action.
Hence, Ds can receive their share of current firm income and a return
of their capital contributions.
■ c. The burden of proof is placed on Ds to show that the clients who
left would have consented to leave even if there had not been a breach
of the fiduciary duty. If Ds cannot met their burden, they must
account to the firm for any profits they received from these cases, plus
23
they must pay the firm the fair charge called for by the partnership
agreement for removing the client
○ Limited Partnerships
■ 1. Differences from General Partnership
● a. Formalities: Need to file certificate of limited partnership (A general partnership
can be formed orally)
● b. Two categories of partners: GP & LP
● c. Limited liability for LPs; Unlimited for GP
● d. Management in GP; LPs are passive investors
● e. Profit and Loss Sharing: LPs share in profits and losses based on their
contributions.
● f. Dissolution: Dissociation of LP does not dissolve
○ i. Holzman v. De Escamilla
■ A. Limited Partners shall not be liable as general partners unless, in
addition to the exercise of his rights and powers as a limited partner,
he takes control of the business.
■ B. The court held that the LPs sufficiently controlled the business
over the GP to be liable as GPs.
■ 2. RUPA §303(a): Limited partner is NOT LIABLE FOR OBLIGATIONS of limited
partnership unless:
● a. Limited partner is also a general partner OR
● b. Limited partner takes part in the control of the business (DeEscamilla) In this case,
limited partner is liable to 3P who transact business with limited partnership and who
reasonably believe on limited partner’s conduct that she is a general partner.
■ 3. RULPA §303(b): Limited partner does not participate in control solely by
consulting/advising with general partner on partnership business.
● Corporations
○ Comparisons of Corporations and Partnerships
■ PARTNERSHIP
● Informal
● Unlimited liability
● Not freely transferable
● At will
● Equal management rights
● Single taxation
■ CORPORATION
● Formalities required
○ The government requires certain particular findings.
● Limited liability
○ Default rule, although the veil may be pierced in certain circumstances.
● Free transferability
○ Shares can be freely bought/sold. You can freely transfer your ownership
interest in corporations. You can sell your shares. This is different if you
were a partner in a partnership. The default rule in a partnership is that you
can’t transfer unless you receive consent.
24
● Continuity across management
○ If someone dies in a partnership, what is the general rule? The partnership
folds. It is supposed to be wound up. This means that a partnership is only
good as long as the original partners survive.
○ The corporation has a life separate from the original members. A
corporation has legal personality. It can sue and be sued. It can hold
property in its own name. So, again, it is in theory supposed to exist in
perpetuity.
● Centralized management
○ Clearly set out boards
○ In a large business corporation, shareholders delegate their authority to
manage to a professional gorup of managers.
○ In a closely held corporation, the shareholders are also officers.
○ In a partnership, every partner has equl rights in management (default rule
in §18)
● Double taxation
○ On business and capital gains. If the profits of a company are paid out to
shareholders in a distribution called a dividend, the individual shareholders
pay income tax as well – Double taxation does not exist in partnership.
○ Setting up a corporation
■ (See incorporation packet and notes)
■ Choice of corporate form
■ Choice of state of incorporation
● Can choose any jurisdiction, even though not the state where business has principal
office
● Internal affairs doctrine
● Focus on Delaware General Corporation Law
○ We’re in Missouri. Why are we studying the law of Delaware in this course?
■ Reserve corporate name
■ Draft and file certificate of incorporation § 102
● Mandatory
○ Name (include the words Inc. or Corp)
○ Address
○ Business/Purpose (any lawful business)
○ Capitalization structure (shareholders have identical rights unless specified
otherwise)
○ Incorporators’ names and addresses
○ Directors’ names and addresses
● Optional
○ Management provisions/provisions limiting powers of corporation, directors,
shareholders
○ Preemptive shareholder rights
○ Provisions changing the voting rules of DGCL
○ Limit on duration of business
○ Exceptions to limited liability of shareholders
○ Limits on monetary damages for director breach of fiduciary duty (some
fiduciary duties cannot be eliminated)
● Requirement of filing
25
○ §101(a) File with Division of Corporations in the Department of State
○ Filed documents are a matter of public record
■ Hold first meeting of directors to adopt by-laws, issue shares for consideration & take other
action (if directors designated in certificate; if not hold meeting of incorporators)
■ Issue shares and accept paid in capital
■ Qualification as a foreign corporation in all states where doing business
○ Incorporatrors
■ §101 – any person may incorporate a corporation by filing certificate with Division
of Corporations of Secretary of State
■ §103 – signed & dated, pay fees
■ §107 – if no directors named in certificate, incorporators manage business until
directors elected
■ Distinguish roles of shareholders, directors, & officers in the corporation.
○ Commencement of Corporate Existence
■ §106 – corporation exists from the date of filing until dissolution
○ Registered Office
■ DGCL §131 – Registered office required; may or may not be place of business
■ Receives service of process within the state
■ Must be a resident person or corporation
○ By-laws
■ §108 – By-laws adopted at organization meeting of directors or incorporators
■ §109 – May contain provisions on business, conduct of affairs, rights or powers of
shareholders, directors, officers, employees
● May be amended by directors until payment of initial capital; after this, the
shareholders must vote to amend (subject to contract)
● Not filed with the Secretary of State.
○ Promoters and The Corporate Entity
■ Promoter = one who takes the entrepreneurial initiative in founding a business that becomes
a corporation
■ Areas of legal concern:
● Fiduciary duty
○ Owed by a promoter to a corporation which is going to come into existence.
The promoter has to put the interest of the corporation ahead of his own
personal interest.
○ Self-dealing: not acting in the benefit of other promoters or corporation, but
for self.
■ 1. There is no clear cut approach, but may be find liable under
agency law.
○ Promoter has status akin to joint venturer or partner.
○ Duties owed among promoters and to corporation to be formed.
● Can the corporation become party to the Contract?
○ Yes, if the corporation takes significant steps to enter into the contract as a
corporation. Also through implicit action.
● Promoter’s liability before formation of the corporation:
○ When does the corporation become liable on the K, and can a promoter be
personally liable for a pre-incorporation K?
■ 1. Under agency law, the general rule, is that promoter is liable on
pre-incorporation Ks unless corporation adopts the K.

26
■ 2. This can be changed by contract when corporation enters into the
contract.
○ Defective Incorporation
■ Corporation by estoppel
● Person acted as though he was dealing with a corporation
● Would earn a windfall if allowed to evade liability based on absence of incorporation
● Test used in Southern Gulf Marine v. Camcraft: were substantial rights affected?
■ De Facto Corporation (Southern-Gulf Marine Inc. v. Camcraft Inc.)
● Promoters tried in good faith to incorporate
● Had a legal right to do so
● Acted as a corporation
○ (Situations where someone tried in good faith to incorporate, had a legal
right to do so and acted as a corporation, but some problem not attributable
to the person, the entity is not incorporated.)
■ Southern-Gulf Marine Inc. v. Camcraft Inc.
● Facts: Before Southern-Gulf (P) was incorporated, its president entered into a K
with Camcraft (D). Camcraft (D) sought to avoid the K on the theory that Southern-
Gulf Inc. never became liable on the K as it was not incorporated at the time the K
was entered into.
● Holding: Defendant estopped from denying the corporate existence of the plaintiff.
● Analysis: (Corporation by estoppel doctrine) If a party contracts with an entity it
acknowledges to be and treats as a corporation, and incurs obligations in its favor,
and is later sued for performance, it is generally estopped from arguing lack of
corporate existence as a defense. This principal comports with the rules of construing
contracts so as to give them effect and of interpreting contracts in accordance with
justice and fair dealing, with any doubts resolved against the seller.
● Caveat – unless the substantial rights of the party were affected by non-
incorporation
○ Person acted as though he was dealing with a corporation, and he would earn
a windfall if allowed to evade liability based on absence of incorporation.
○ The Corporate Entity and Limited Liability
■ A corporation has legal “rights” and “duties” as a separate legal entity. The owners
(shareholders) have limited liability; debts and liabilities incurred by the corporation belong
to the corporation and not to the shareholders.
■ Theories of holding a corporation liable
● Enterprise liability: Treat all corporation as one. To satisfy recovery, assets of other
corporations owned by the corporate defendant are reached. Disregard multiple
incorporation's of the same under common ownership. This is not personal liability.
● Piercing the corporate veil: Courts sometimes disregard the rule of limited liability to
hold shareholders, directors and officers personally liable for corporate
organizations. Shareholder’s personal assets may be available to creditor.
● Only really comes up with small businesses
○ Judicially created exception to the legal rule of limited liability
○ Factors
■ Unity of interest (alter ego)
● Example: Van Dorn test (Illinois): (Sea Land)
○ Failure to maintain corporate formalities
○ commingling of assets/funds,
○ undercapitalization
27
○ one corporation treats assets of another as its own
■ Fraud or injustice; need more than creditor’s inability to collect
● Undercapitalization as a factor in piercing
○ Shareholder siphoning off available corporate assets without disclosure to
creditors, such that corporation is deliberately made insolvent, justifies
piercing in some cases.
○ Deliberate insolvency defeats creditor’s expectation that business will set
aside adequate reserves to pay corporate obligations when due.
■ Example: Flemming was the sole shareholder of a fruit brokerage
business in which he acted as a middleman between growers and
buyers. He collected the sales price from the buyers, deducted his
commission and the transportation charges, and sent the balance to the
growers. However, he also paid himself a salary from time to time
that included the amount withheld from the growers’ payment to take
care of transportation charges. When truckers sued for unpaid bills,
he claimed insolvency. Court pierced and held Flemming personally
liable. De Witt Trucking v. Flemming (4th Cir. 1976)
● Assumption of risk in contract cases
○ Some courts impose a third prong in contract cases: assumption of risk
■ Did the creditor know of the risk of nonpayment? Creditor should
have taken steps to mitigate the risk.
■ Example: Shareholder set up a corporation with no assets solely to
make payments under a supply contract. Corporation defaulted.
Court refused to pierce since supplier knew the facts and was not
misled. Supplier could have taken steps to protect itself, like
demanding a personal guarantee, but failed to do so. Supplier
assumed the risk of loss. Brunswick Corp. v. Waxman (2d Cir. 1979)
■ Wallovsky v. Carlton
● Facts: P was severely injured in a taxicab accident. P sued the cab driver, the
corporation owning the cab, and Carlton (D), who owned that corporation and nine
others, each corporation having two cabs with the minimum $10,000 liability
insurance. The complaint asserted that the corporations operated as a single entity
and constituted a fraud on the public. D moves for dismissal, asserting that P had not
alleged a cause of action.
● Holding: Plaintiff did not show evidence that the corporation was being carried on
for personal/individual capacity rather than corporate purposes. Therefore, the court
did not pierce the corporate veil.
■ Sea-Land Services, Inc. v. Pepper Source
● Facts: Plaintiff delivered a shipment of peppers for Pepper Source, but they were
not paid. Marchese was the sole shareholder of Pepper Source. Marchese was also
the sole shareholder of several other corporations, and he was a co-owner of an
additional corporation. Plaintiff asserted that the corporations were shells wherein
Marchese shifted money around the different entities to avoid creditors collecting
from the corporations. Evidence was presented that showed Marchese treated the
corporate accounts as his own personal account, and he frequently shifted money

28
around. Plaintiff wanted to pierce the corporate veil to hold Marchese liable, and
reverse pierce Marchese’s other corporations.
● Holding: The corporate veil was pierced because the court could not differentiate
between the corporation and Marchese. Protecting Marchese would unjust.
● Analysis: A corporate veil may be pierced if two requirements are met (Van Dorn
test):
○ 1) Unity of interest and ownership that the separate personalities of the
corporation and the individual no longer exist; and
■ (Factors)
● 1) A failure to comply with corporate formalities or to keep
sufficient business records
● 2) A commingling of corporate assets
● 3) Undercapitalization, and
● 4) One corporation’s treatment of another corporation’s assets
as its own.
○ 2) Circumstances must be such that adherence to the fiction of separate
corporate existence would sanction a fraud or promote injustice. The
“wrong” must be beyond a creditor’s inability to collect.
■ Examples: the common sense rules of adverse possession would be
undermined; former partners would be permitted to skirt the legal
rules concerning monetary obligations, a party would be unjustly
enriched, a parent company that caused a sub’s liabilities and its
inability to pay for them would escape those liabilities
○ 3) (SOME COURTS) Prong in contract cases- assumption of risk.
■ Defense: Party was aware of a risk and nonethless went through with
the transaction.
■ R.C. Archbishop of San Francisco v. Sheffield (liability between subsidiaries)
● Facts: Sheffield entered into an agreement with Fr. Cretton to purchase a dog at a
monastery in Switzerland, payable in $20 dollar installments and totaling $175. The
dog was never sent, so Sheffield sued the Archbishop of San Francisco, alleging that
because Archbishop of San Fran, the Pope, the Vatican, and the monastery were alter
egos of one another and therefore could hold the Archbishop liable for the breach of
the agreement.
● Holding: Since the archbishop was not controlled by the monastery, the corporate
veil could not be pierced.
○ Note: even if the unity of interest prong was satisfied, the fact that Sheffield
could not collect a refund would not have satisfied the “promote
fraud/injustice” prong of the Van Dorn test. The purpose is not to protect
every unsatisfied creditor, but rather to afford him protection, where some
conduct amounting to bad faith makes it equitable for the equitable owner of
a corporation to hide behind its corporate veil
● Analysis: The alter ego/pierce the corporate veil theory makes a ‘parent’ liable for
the actions of a ‘subsidiary’ which it controls, but it does not mean that where a
‘parent’ controls several subsidiaries each subsidiary then becomes liable for the
actions of all other subsidiaries.
○ Parent-Subsidiary Relationship

29
■ “Substantial Domination” → Used to pierce the corporate veil (Bristol Myers case)
● Common directors & officers
● Common business departments
● File consolidated financial statements/tax returns
● Parent formation & financing of subsidiary
● Gross undercapitalization
● Payment of salaries and other expenses
● All business of subsidiary provided by parent
● Parent uses property of subsidiary
● Daily operations not separate
● Failure of subsidiary to maintain corporate formalities
■ Direct Liability
● Restatement 2nd of Torts, Section 324A
○ One who undertakes to render services is subject to liability to 3P for
physical harm due to failure to exercise reasonable care if:
■ Failure to exercise reasonable care increased risk of harm
■ Undertaken to perform duty owed by the other to 3P
■ Harm by 3P suffered because of reliance
■ In re Silicone Gel Breast Implants Products Liability Litigation (Parent to subsidiary)
● Facts: Class action Ps sue Bristol (D), sole shareholder and parent corporation of
MEC (co-D), who manufactured and distributed breast implants.
● Holding: Court found sufficient evidence to hold Bristol liable through substantial
domination (for piercing the corporate veil) and direct liability (for tort claims).
● Analysis:
● Parents aren’t usually liable for debts of subsidiaries UNLESS liability can be
adduced to a parent when it exercises “Substantial domination” over another
corporation (See factors above.) → Note, this is for tort cases. Therefore, the need
to show fraud or injustice is not required. (Policy- You willingly enter into and
assume the risk in a contract case. However, in a tort case, you’re more at the mercy
of the corporation.) (Bristol → MEC → Harm from manufacturing through MEC)
● Direct-liability (See above). (Bristol → Harm from direct lack of inspection)
● Shareholder Derivative Lawsuits
○ DERIVATIVE LAWSUIT: SH sues on behalf of corporation to enforce rights of corporation
(Cohen)
■ “Two suits in one”
■ Sues corporation in equity (defendant is a nominal defendant)
■ To bring an action to enforce corporate rights
■ Modern derivative suit is treated as one suit
■ Recovery runs directly to corporation
■ Involve allegations of mismanagement, waste, fraud by corporate officers and directors
○ Attorney’s fees
■ If suit is successful, the corporation will pay plaintiff’s fees and expenses.
■ Some state statutes mandate fee shifting to plaintiff if suit was brought without reasonable
cause or for an improper purpose.
■ Security for fees statutes are now uncommon.
○ DIRECT LAWSUIT:
■ SH suit in personal capacity to enforce rights as a shareholder

30
■ Examples: denial or dilution of voting rights; compel payment of dividends declared but not
distributed; compel inspection of corporate books and records, require holding of a
shareholder meeting
○ CLASS ACTION:
■ SH sues in his own capacity as well as on behalf of other SHs similarly situated
■ Group of SHs assert their individual direct claims through a representative
■ Procedural rules applicable to class actions such as plaintiff must be representative of other
SH interests and settlement must be approved by court
■ Some derivative suit procedural hurdles (like demand) may not apply to class actions but
other procedural hurdles may apply (like giving notice to class members
○ Cohen v. Beneficial Industrial Loan Corp. (Derivative)
■ Facts: Cohen (P), a shareholder filing a derivative action, challenged the constitutionality of
a NJ statute requiring an unsuccessful plaintiff to indemnify the corporation for its
reasonable expenses in defending the action.
■ Holding: The NJ statute should be followed. It is not procedural. It is constitutional.
■ Analysis: Shareholder derivative suits still have to follow applicable state laws. States are
within their rights to determine how they want to resolve competing interests and plaintiffs
can’t avoid state statutes by claiming diversity and using the federal courts.
○ Eisenberg v. Flying Tiger Line, Inc (Direct lawsuit/class action)
■ Facts: Eisenberg sought to overturn a recognition and merger of Flying Tiger, which he
claimed was intended to dilute his voting rights as a stockerholder. The merger transformed
the company in which he held stock into a holding company and the operations of the
original company were transferred to holding company’s subsidiary. Therefore’ he had no
voting rights. Eisenberg argued his suit was a representative in a class action suit.
■ Holding: A suit over the dilution of voting rights is NOT a derivative lawsuit.
■ Lazer test:
● Derivative only if brought in the right of a corporation to procure a judgment in its
favor. This is a suit in equity, filed on behalf of the corporation to enforce corporate
rights, where recovery is running to the corporation.
○ Typically involves claim of mismanagement: the activity provides no benefit
to the corporation (waste) or some type of fraud involved. Mismanagement,
fraud or waste.
● Direct when injury to individual’s right and privileges as a stockholder and recovery
will not go the corporation.
■ Analysis: Suits are derivative only if brought in the right of a corporation to procure a
judgment “in its favor.” A suit based on interference with a plaintiff’s rights and privileges
as a stockholder by directors does NOT constitute a derivative action.
○ Demand Requirement
■ Purpose
● Allow dispute to be resolved by corporation outside of court
● Allow corporation to take over the lawsuit if it is beneficial to corporation
● If demand is excused or wrongfully refused, SH will be allowed to control
proceedings
● Protect corporate boards from harassment and discourage strike suits
○ Demand Futility
■ Delaware
● Allege facts with particularity creating reasonable doubt regarding board
independence
● Either:
31
○ Majority has material financial or familial interest, OR
○ Majority is incapable of acting independently for another reason like
domination or control, OR
○ Underlying transaction is not the product of a valid business judgment
■ New York
● Allege facts with particularity that either:
○ Majority of the board is interested in the transaction, OR
○ Directors failed to inform themselves as reasonably necessary about the
transaction, OR
○ Directors failed to exercise their business judgment in approving the
transaction
○ Wrongful refusal
■ Delaware
● Board decision on demand is protected by the business judgment rule
● To overcome, plaintiff must allege facts with particularity creating reasonable doubt
that board acted independently or with due care
○ Grimes v. Donald (Demand requirement)
■ Facts: Grimes (P), a shareholder, unsuccessfully sought a declaration of the invalidity of
certain agreements made between Donald (D), the CEO, and the Board of Directors of DSC
Communications Corp, alleging excessive compensation and abdication of directional duty
by the Board of Directors of the corporation.
■ Holding: Grimes’ complaint failed. If a demand is made and rejected, the board rejecting
the demand is entitled to the presumption of the business judgment rule unless the
stockholder can allege facts with particularity creating a reasonable doubt that the board is
entitled to the benefit of the presumption.
■ Analysis: If you make demand and it is rejected, you have to allege facts with particularity
why the board’s refusal to act was wrongful. If you make demand and it is rejected, you
can’t go back and argue that demand was excused.
● The basis for claiming demand excusal would normally be that:
○ 1) a majority of the board has a material financial or familial interest;
○ 2) a majority of the board is incapable of acting independently for some other
reason such as domination or control; or
○ 3) the underlying transaction is not the product of a valid exercise of business
judgment
○ Marks v. Akers (Demand futility excpetion)
■ Facts: Plaintiff filed shareholder derivative lawsuit against board of directors alleging
corporate waste for officers’ excessive compensation.
■ Holding: Demand was excused because the directors were interested parties, however, P
failed to allege facts with particularity that showed why the compensation was excessive.
■ Analysis/Rule: Demand is excused because of futility when a complaint alleges with
particularity: (1) that the majority of the board of directors is interested in the transaction
(Director interests may either be self-interests or a loss of independence because a director
with no direct interest in a transaction is controlled by a self-interested director); (2) board
didn’t inform themselves about the transaction to the extent reasonably appropriate under
the circumstances; or (3) the challenged transaction was so egregious on its face that it could
not have been the product of the directors’ sound business judgment.

32
● A director will always be an interested party, for the purpose of the excusal of a
demand, when the director is voting on director compensation, but a plaintiff has to
demonstrate with particularity that the compensation is excessive
● Special Litigation Committees
○ Special litigation committees are often appointed of new, disinterested directors to decide whether
the corporation should proceed with the litigation.
○ Board Committees
○ Permitted under DGCL §141 (C)(2)
■ Board may designate 1 or more committees, each consisting of 1 or more directors
■ Board committee may exercise all the powers and authority of the full board
○ Business Judgment Rule
■ DGCL, §141(a) – business and affairs of corporation shall be managed by or under the
direction of the board of directors
■ Rebuttable presumption that directors and officers carry out their functions in good faith,
after sufficient investigation and for valid business reasons
○ Aurbach v. Bennett (Special Litigation Committees New York)
■ Facts: Aurbach, a shareholder, brought a dirivative suit on behalf of corporations. Board of
directors created a special litigation committee composed of disinterested directors, which
concluded the action shouldn’t proceed.
■ Holding: The creation of a special litigation committee of disinterested directors (even if
appointed by interested directors) was proper. Court may inquire into the
procedure/investigative methods of the board, but will not inquire into the substantive
decisions (which are protected by the business judgment rule) of the board to proceed or
dismiss the suit.
● A party may challenge the independence of a special committee, but once a
committee is deemed to be independent then their decisions are protected under the
business judgment rule
○ Zapata Corp v. Maldonado (Special Litigation Committees- Delaware)
■ Facts: Maldonado was a shareholder of Zapata and initiated a derivative suit alleging breach
of fiduciary duty against certain officers and all the directors. He did not make demand,
claiming demand futility because all directors were named as defendants. After replacement
of some board members, the new board created an investigation committee. The committee
determined that each action against the corporation should be dismissed.
■ Holding: The special investigation committee’s decision to dismiss was not protected by the
business judgment rule. Needed to inquire into independence to committee.
■ Analysis: In Demand excusal cases use 2 step inquiry:
● 1) The court must recognize that the board, even it fainted by self-interest, can
legally delegate its authority to a committee of disinterested directors. The court may
inquire on its own into the independence and good faith of the committee and the
bases supporting its conclusions – must be reasonable bases. Burden on corporation
to prove independence, good faith and reasonable bases.
● 2) If the court is satisfied, the second step is to apply its own business judgment as to
whether the motion to dismiss the lawsuit should be granted. (A higher standard
since it allows the court to evaluate the decision’s validity)
○ In re Oracle Corp. Derivative Litigation
■ Facts: Derivative lawsuit-according to P, Defendants’s possessed material, non-public info
demonstrating that Oracle would fail to meet the earnings and revenue guidance it had
provided to the market in Dec 2000. Oracle formed a Special litigation Committee.
33
Company set up a SLC w/ two members from Stanford. Advisors retained to perform
analytical work to demonstrate good faith. SLC concluded that Oracle should not pursue the
P’s claims against the TD’s or any of the other Oracle directors.
■ Holding: SLC has the authority to do this.(procedure ok, but ct has problem w/ SLC’s
independence (Committee’s ties to Stanford)
● (i) Independence-B/c the ties among the SLC, the TD’, and Stanford are so
substantial that they cause reasonable doubt about the SLC’s ability to impartially
consider whether the TD’s should face suit
● (ii) DEL LAW- independence of the special litigation committee
○ 1. Test: Law turns on economic ties, or domination and controlàtends
toward finding in favor of D
■ a. In this case, the ct, by looking at human nature is expanding the
test. Even where you don’t have an economic interest or repercussion,
you may not be dominated or controlled by someone w/ an economic
interest. The fact that you have social ties and a strong friendship may
tip the balance away from the SLC committee’s independence
■ b. We don’t look @ special litigation members as a special kind of
individual who only has economics as a motivating factor
■ You have to look @ human nature-such as personal feelings that
they have towards the D which would keep them from making
decision contrary to the D’s interest
■ Problem in this case was that SLC didn’t first reveal all their ties,
when the ties were discovered à it was shocking
● (iii) Note--Subsequent development
○ 1. Even though independence is the special focus of inquiries there is
difference on where the burden of proof lies
○ a. Under BJ rule, presumption that they are independent
○ b. Under SLC, burden is shifted to the board to prove they are independent
○ ·Where Oracle’s special litigation committee falls apart
● (iv) BOARD HAS BURDEN
● The Role and Purpose of Corporations
○ Corporate Powers and Ultra Vires Act
■ DGCL §102(A)(3)
● Certificate of incorporation shall set forth the nature of the business or purposes to be
conducted or promoted.
● May simply say “any lawful act or activity”
● May contain restrictions
■ DGCL §124
● No act or transfer of property shall be invalid because ultra vires but lack of capacity
or power may be asserted:
● Shareholder suit to enjoin corporation from entering into such act or transfer of
property
● Corporate suit against directors and officers
● Suit by state attorney general
○ Charitable Donations
■ DGCL §122
● Every corporation shall have the power to … sue and be sued, … acquire real or
personal property and dispose of same, … conduct its business within or without this

34
state, … appoint officers, …wind up and dissolve, … make donations for the
public welfare or for charitable, scientific or educational purposes, … make
contracts and borrow/lend money, … pay pensions, … buy insurance for its benefit
on life of directors, officers, employees, or any shareholder …
■ California Corporations Code Section 207(3): power to make donations regardless of
specific corporate benefit for the public welfare, or for community fund, hospital,
charitable, educational, scientific, civic or similar purposes
■ New York BCL Section 202(a)(12): make donations irrespective of corporate benefit for
the public welfare or for community fund, hospital, charitable, educational, scientific, civic
or similar purposes, and in time of war or other national emergency in aid thereof
■ With Delaware law, have to show a specific benefit (charitable, scientific or educational
purposes), while with CA and NY, do not need to demonstrate benefit. (Therefore,
anonymous gift has more problems in Delaware b/c corp. needs to show benefit)
○ Smith v. Barlow (Charitable contributions)
■ Facts: Barlow (D) and other shareholders of AP Smith Mfg. Co. (P) challenged its authority
to make a charitable donation to Princeton University. Shareholder challenges this conduct
based on ultra vires, that is, the board was not granted this power—it is no legal basis for
this power.
■ Holding: Corporations can make charitable contributions without authorization of
stockholders or express authority in the articles of incorporation if the contributions are
voluntarily made in the reasonable belief that it would aid the public welfare and advance
the interests of the company as a private corporation and as part of the community in which
it a operates
■ 1) Limitations on Gift Giving (Smith)
● (a) Has to be a corporate benefit that flows from it
○ (i) Can be tangible or intangible benefit (PP)
■ 1. Good will of the company and enhancing corporate prestige-
Smith
■ 2. Building corporate business
■ 3. Could end up recruiting employees
■ 4. Further democratic values and whole country will benefit
○ (ii) Anonymous gift-may create some problems under Del law
● (b) Has to a reasonable amount- unreasonable amount-related to the financial size of
the company
● (c) It can’t be to a pet charity-a charity favored by the managers
○ Payment of Dividends
■ DGCL Section 170(a): Directors may declare and pay dividends out of surplus or net
profits, subject to restrictions in certificate of incorporation.
○ Dodge v. Ford Motor Co.
■ Facts: Shareholders Horance and John Dodge (P) filed suit against the Ford Motor Co (D)
after its majority shareholder, Henry Ford, decided NOT to pay any more special dividends
to shareholders but instead to reinvest the money in the business.
■ Holding: Court takes a narrow view of the purpose of the company which is to make money
for the shareholders. This is a dying view. Many have criticized this. (Idea of brand loyalty

35
might have been good for Ford if they made it more affordable. More pleased and
productive workforce.)
■ Old view: Hunter v. Roberts (Michigan SCt): “It is a well recognized principle that
directors alone have power to declare a dividend and determine its amount. Courts of equity
will not interfere in the management of the directors unless…they are guilty of fraud or
misappropriation of the corporate funds, or refuse to declare a dividend when the
corporation has a surplus of net profits…and when a refusal would amount to abuse of
discretion as would constitute fraud or breach of good faith…”

■ Modern view: charitable giving, cleaning up environmental pollution, paying people better
wages, and having better working conditions are considered valid uses of corporate funds.
○ Shlensky v. Wrigley
■ Facts: Wrigley (D), the majority shareholder in the Chicago Cubs, refused to install lights at
Wrigley Field in order to hold night games, and Shlensky (P), a minority shareholder, filed a
derivative suit to compel the installation.
■ Holding: The court will not disturb the business judgment rule without allegations of fraud,
illegality, or conflict of interest.
● Courts may decide questions of negligence and wrongful conduct by directors when
there is a showing of dereliction of duty on the part of the specific directors. Mere
failure to “follow the crowd” is not such a dereliction.
○ Berle-Dodd Debate
■ Berle took a private property view of corporation (expressed in Ford Motor Company) –
purpose is solely to benefit the shareholders, and managers shouldn’t do anything contrary
to this goal
■ Dodd took a social institution view in which corporations served larger community interests
– corps. should take into account employees, customers, etc
○ The New Corporate Social Responsibility
■ Corporate Social Responsibility = companies voluntarily decide to respect and protect the
interests of a broad range of stakeholders and to contribute to a cleaner environment and
better society through active interaction with all
■ Supported by company level, industry level, and international codes of conduct
● Developed by business, civil society, national governments and intergovernmental
organizations
■ Areas of coverage
● Core labor standards
● Green environmental standards
● Reject bribery and corruption of government officials to facilitate business
● Respect human rights
● Limited Liability Company
○ Introduction
■ “Hybrid” = Can combine limited liability and flow through tax treatment
■ Created under state statutes enacted in all 50 states
■ ULLCA provides rules for structure, governance, and operation
■ ULLCA §201: LLC is a legal entity distinct from its members
■ Corporate Characteristics
● Limited liability
● Free transferability
36
● Continuity of life
● Centralized management (versus member management)
■ Tax treatment:
● Prior to 1997, LLC could only receive flow through tax treatment if it had 2 or fewer
of the above attributes
● IRS 1997 Check the Box regulations simplify tax treatment of LLCs
○ Documents Required for Formation
■ Filing of Articles of Organization with state
● Contents are dictated by statute
■ Execution by members of Operating Agreement
● Not filed with state and not publicly available
● Often cover topics such as membership, governance, finance, dissolution
● Flexible structure can be used to revise default rules subject to limits spelled out in
ULLCA §103
○ Formations: Articles of Organization
■ ULLCA §202: One or more persons (“organizers”) may form LLC, consisting of one or
more members, by filing articles with secretary of state
■ LLC comes into existence when articles filed with secretary of state
■ ULLCA §203: Contents of Articles of Organization
● Name of company
● Address
● Name and address of agent for service of process,
● Name and address of organizer
● Term, if there is one
● Whether manager managed and name and address of managers
● Liability of members for debts and obligations, if applicable
■ ULLCA §204: Amendment of Articles of Organization
● Must be filed with state
○ Formation Operating Agreement
■ Notice Provision of LLC Act (Westec)
● “The fact that the articles of organization are on file in the office of the secretary of
state is notice that the limited liability company is a limited liability company and is
notice of all other facts set forth therein which are required to be set forth in the
articles of organization.”
■ Formation-Operating agreement
● ULLCA §103(a): Members of LLC may enter into Operating Agreement to regulate
conduct of business and relations among members, managers and company
○ May not change provisions of the ULLCA specified in §103(b)
■ The operating agreement may not:
● (1) unreasonably restrict a right to information
● (2) eliminate the duty of loyalty
● (3) unreasonably reduce the duty of care
● (4) eliminate the obligation of good faith and fair dealing but
can determine the standards by which it is measured as long as
they’re long unreasonable.
● (5)-(7)
○ Unless changed by Operating Agreement, the default rules of the ULLCA
apply
■ Westec v. Lanham
37
● Facts: Westec (P) sought to hold the managers, Lanham (D) and Clark (co-D), of
Preferred Income Investors (D), an LLC, personally liable for the company’s failure
to tender payments for services performed. Defendant gave business card with name
and business abbreviation.
● Holding: To avail oneself of liability, must put 3rd party on constructive notice that
you’re an LLC. Notice would be placing LLC with company name.
○ “If both the existence and identity of the agent’s principal are fully disclosed
to the other party, the agent does not become a party to any contract which he
negotiates…But where the principal is partially disclosed (i.e. the existence
of a principal is known but his identity is not), it is usually inferred that the
agent is a party to the contract.”
■ Elf v. Jaffari
● Facts: Elf (P) and Jaffari (D) agreed to undertake a joint venture that would be
carried out by forming a LLC. Elf sued Jaffari for damages arising out of alleged
breaches of fiduciary duty. LLC had forum selection clause in operating agreement.
● Holding: Parties may, in their operating agreement, K to avoid the applicability of
certain provisions of the act, as long as the agreement does not contain any of the
prohibited clauses/terms in ULLCA § 103(b).
○ Piercing the “LLC” Veil
■ ULLCA §303
● Debts, obligations, liabilities of LLC belong solely to the company
● Members or managers are NOT personally liable for debts, obligations and liabilities
of LLC, whether arising in contract, tort or otherwise
● Failure to observe formalities or requirements for exercise of powers or management
are NOT a ground for imposing personal liability
● Members can become LIABLE for debts etc. if they so consent in writing and
include a provision in articles
■ Kaycee v. Flahive
● Facts: Flahive Oil & Gas of Wyoming-Limited Liability Company w/ no assets.
Kaycee Land and Livestock entered into a K w/ Flahive to use the surface of its real
property. Roger Flahive-managing member of Flahive Oil & Gas. Kaycee alleges
that Flahive caused environmental contamination to its real property located in
Wyoming. (no alleged fraud). Kaycee seeks to pierce the LLC veil and disregard the
LLC entity of Flahive Oil and hold Roger Flahive individually liable for
contamination
● Holding: A court may pierce an LLC veil in the same manner as court would pierce
a corporate veil. Follow state LLC statute.
● Minnesota LLC Statute
● “case law that states the conditions and circumstances under which the corporate veil
of a corporation may be pierced under Minnesota law also applies to limited liability
companies” Minn. Stat. §322B.303(2)
● Minnesota Case Law on Corporate Piercing
● “Courts will pierce the corporate veil if (1) an entity ignores corporate formalities
and acts as the alter ego or instrumentality of a shareholder and (2) the liability
limitations of the corporate forum results in injustice or is fundamentally unfair.”
Victoria Elevator Co., Inc. v. Meriden Grain Co., 283 N.W.2d 509, 512 (Minn.
1979) (listing several factors supporting alter ego or instrumentality finding). "The
practice of piercing the corporate veil is generally a creditor's remedy used to reach
an individual who has used a corporation as an instrument to defraud creditors." Id.
38
○ Fiduciary Duty
■ ULLCA §409: General Standards of Member’s and Manager’s Conduct
● Members owes duty of loyalty and care to member managed company and its other
members
● Duty of loyalty: accounting for profit from use of property of LLC and
appropriation of corporate opportunity, refrain from acting adversely to the LLC’s
interests, refrain from competing with the company in the conduct of its business
before dissolution
● Duty of care: do not engage in grossly negligent or reckless conduct, intentional
misconduct, knowing violation of law
■ McConnell v. Hunt (fiduciary duties)
● Facts: McConnell (P) and Hunts Sports Enterprises (D) formed an LLC for the
purpose of investing in and operating an NHL franchise. They had written into their
agreement that members could compete against the LLC. The (D) acquired the
franchise outside of the LLC.
● Holding: Members of an LLC can agree to limit the scope of the fiduciary duty they
owe to the LLC.
○ Styles of LLC management
■ There are two styles of LLC Management: Member management and Manager management
■ ULLCA §404
■ Member managed: Members have equal rights in management, majority voting for
business decisions
■ Manager managed: Managers have equal rights in management, majority voting by
managers for business decision
■ Managers must be designated, removed, replaced by a majority of Members
■ Consent of all Members required for certain important events
○ Agency of Members and Managers
■ ULLCA §301
● Each Member is an AGENT of LLC for purpose of its business and acts of Members
for apparently carrying on ordinary business of LLC binds the company, unless there
was no authority and 3P knew this
● Acts not for apparently carrying on ordinary business binds only if authorized by the
other members
■ In a manager managed LLC:
● A member is not an agent solely by reason of being a Member each manager is an
agent and binds the company if apparently carrying on ordinary business of LLC,
unless the manager had no authority to act on that matter and 3P had notice of that
fact.
● Act of a manager not for apparently carrying on ordinary business binds company
only if authorized as required by ULLCA §404
○ Dissolution under ULLCA
■ §801 – Events of dissolution
■ §802 – Winding up after dissolution
■ §805 - Filing of articles of termination
■ §807 – Disposing of known claims by giving notice to creditors
■ §808 – Disposing of other claims by publishing notice to creditors in a newspaper
■ Other claims are barred

39
■ §806 – Creditors must be paid! (including Members); Members entitled to return of
contributions
■ §808(d)(2) – Member liability to creditors up to amount received in distribution
■ New Horizons v. Haack
● Facts: When New Horizons Supply Cooperative (P) stopped receiving payments
from Allison Haack (D) for fuel it had sold to her, New Horizons (P) sued Haack (D)
for the remaining balance.
● Holding: Members of an LLC can be held personally liable for the debts of the LLC
if they fail to properly disolve the LLC.
○ The proper dissolution of an LLC is one where the statutory requirements are
not flexible. The requirements are in place to insure that 3rd parties are
protected when dealing with the LLC.
○ An LLC is terminated ONLY upon the filing of article of termination [see §
805] –
■ failure to do so results in the ineffective shielding of the members or
managers of an LLC from personal liability
● Corporations: Fiduciary Duty
○ Merger Approval Procedures
■ DGCL §251(b) – BOARD APPROVAL
● Board of each merging corporation shall adopt a resolution approving an agreement
of merger and declaring its advisability
■ DGCL §251(C) – STOCKHOLDER APPROVAL
● Merger agreement shall be submitted to stockholders for approval
● Majority of shares entitled to vote must approve
● If approved by stockholders, merger agreement (or certificate of merger) is then filed
and becomes effective
■ Cash out merger
● One type of corporate combination
● Acquiring company pays shareholders of target company the value of their shares
● Target company is merged out of existence and shareholders have no interest in any
company that results from the merger
● Shareholders protected by fiduciary duties of directors

○ Management by Board of Directors
■ DGCL §141(a) - BOARD AUTHORITY
● Business and affairs of corporation shall be managed by or under the direction of a
board of directors
■ DGCL §141(b) – BOARD COMPOSITION & ACTION
● Composed of one or more members (fixed in bylaws or certificate)
● Majority of total # = QUORUM (can reduce to 1/3 in bylaws unless certificate
provides otherwise)
● VALID BOARD ACTION = vote of a majority of directors present at a meeting
with a quorum present (can require supermajority in certificate or bylaws)
○ Shareholder Voting
■ DGCL §216
● Majority of shares entitled to vote shall constitute a quorum at a stockholder meeting
(can reduce to 1/3 in certificate or bylaws)
● Vote of majority of those present or represented by proxy shall be the act of the
stockholders (except for election of directors)
40
● Directors elected by plurality
○ Kamin v. American Express Co.
■ Facts: Kamin (P) brought a shareholders’ derivative suit claiming that American Express’
(D) decision to declare certain dividends to be in kind was a negligent violation of directors’
fiduciary duty of care.
■ Holding: A director of a company cannot be charged with negligence without alleging fraud
or bad faith. Whether or not a dividend is to be declared or a distribution of some kind
should me made is exclusively a matter of business judgment for the Board of Directors.
○ Good Faith Reliance on Records and Reports (Smith v. Van Gorkom)
■ DGCL §141(e) – In performing their duties, board members may rely in good faith on
records of corporation and on information, opinions, reports, statements presented to the
corporation by corporation’s officers or employees
○ Smith v. Van Gorkom
■ Facts: Sr. Mgmt. & VG discuss sale/leveraged buy out (LBO) of TransUnion; CFO & COO
ran the numbers at $50 & $60/share. VG meets w/ Pritzker & offers him TransUnion at
$55/share, which was an arbitrary number. Pritzker expresses interest; subject to financing,
lock-up option & board approval within 3 days. VG convened meeting of senior
management at 11 AM followed by board meeting at noon. No documentation or valuation
study presented to board. Board approved after 2 hour meeting subject to conditions:
● Reserve right to accept better offer during market test period
● Share proprietary information with other potential bidders
■ Board meeting convened and approved proposed amendments to Merger Agreement sight
unseen; Salomon Bros. Retained to solicit offers for 3 month market test period.
Shareholders approve Pritzker merger proposal. Shareholders sue for rescission and
damages against directors.
■ Holding:
● Delaware Business Judgment Rule, “is a presumption that in making a business
decision, the directors of a corporation acted on an informed basis, in good faith and
in the honest belief that the action taken was in the best interests of the company.”
○ Burden is on plaintiff to overcome the presumption.
● The standard for the duty of care for mergers is that they act in an informed and
deliberate manner in an informed and deliberate manner in determining whether to
approve an agreement of merger before submitting the proposal to the stockholders
(use this standard for the process in §251(b)).
● Shareholder ratification didn’t work here because directors failed to disclose all
material information that a reasonable shareholder would consider important in
deciding whether to approve an offer.
○ Limits on Director Liability
■ DGCL, §102(b)(7)
● Certificate of incorporation may contain provision eliminating or limiting personal
liability of director for monetary damages for breach of fiduciary duty
● EXCEPTIONS
● Duty of loyalty
● Lack of good faith, intentional misconduct, knowing violation of law
● Violation of Section 174 for unlawful payment of dividends
● Transactions in which director derived improper personal benefit
■ Francis v. United Jersey Bank (Duty to Monitor case)

41
● Facts: Lillian Pritchard (D) ignored her duties as a director, allowing her sons to
withdraw over $12 million from client trust accounts. Plaintiffs, trustees in
bankruptcy, sued her estate alleging that she was negligent in allowing her sons to
withdraw funds for own use.
● Holding: Mrs. Pritchard was individually liable for the acts of the corporation.
● What should Pritchard have done to discharge her duty to corporation?
○ New Jersey law requires good faith and due care
○ Directors owe the degree of care that an ordinarily prudent bussinessman
would exercise in the management of their business
○ Must understand business of corporation
○ Must stay current with corporate activities
○ May rely on lawyers and accountants BUT
■ Must inquire further if financial statements disclosed problems on
their face
■ Must object and take corrective action if an illegal course of action is
discovered
■ May have to resign to avoid liability if corp. does not correct the
conduct
● A director who is present at a board meeting is presumed to
concur in corporate action taken at the meeting unless his
dissent is entered in the minutes of the meeting or filed
promptly after adjournment
○ Conflict of Interest Transactions
■ NYBCL §713 – Covers contracts between 1) corporation and 2) director or corporation in
which director has substantial financial interest or is a director or officer
■ NYBCL §713 – No contract involving an interested director (definition) is void or voidable
because director was present/voted in favor if:
● Disclosure and valid board approval without counting vote of interested director
● Disclosure and contract approved by shareholders
● In other cases, must show fair and reasonable
■ Bayer v. Beran
● Facts: Directors of Celanese Corp (D) engaged in an advertising campaign in which
the president’s wife was spokesperson. Shareholders brought suit, alleging a duty of
loyalty and/or conflict of interest violation.
● Holding: The defendant did not breach a fiduciary duty to the corporation because
the advertising campaign ultimately benefited the corporation rather than the singer.
● The mere allegation of conflict of interest triggers this exception to the business
judgment rule (BJR gives way to the duty of loyalty). When a close relative of the
chief executive officer of a company and one of its dominate shareholders take a
position clearly associated with a new and expensive field of activity, the motives of
the directors are likely to be questioned.
● The burden is on the directors not to only show the good faith of the
transaction, but to show inherent fairness to the corporation.
○ Defect in board ratifying procedure not fatal.
■ Directors will not be personally liable for failure to observe formal
procedural requirements if the directors are in daily association with
each other and their full time is devoted to the business of the
42
company with which they have been connected for many years (they
are executives/employees of the corp.)
■ Capital Structure Terminology
● Debt = “an unqualified obligation to pay a fixed sum certain at a fixed maturity date
along with a fixed percentage of interest payable regardless of the debtor’s income or
lack thereof”
● Holders of debt are third party creditors, not owners of the corporation
● Bonds = corporate obligations secured by lien or mortgage on specific corporate
property
● Debentures = unsecured corporate obligations
● Both bonds and debentures are long term obligations; Notes are short term debt
obligations
● In bankruptcy, debt holders have priority in payment over equity holders
● Equity = ownership interest in corporation
○ Common Stock = residual interest in corporate assets after all senior
claimants have been satisfied; voting rights; no fixed dividend
○ Preferred Stock = dividend preference; liquidation preference; usually no
voting rights
○ DGCL Section 151 permits issuance of different classes of stock with varying
voting rights and preferences
○ DGCL Section 102(a)(4) states that the certificate of incorporation must set
forth total number of shares of stock of each class and preferences of each
class
■ Benihana of Tokyo, Inc v. Benihana
● Facts: Aoki owned 100% of BOT stock. BOT owned about 51% of Benihana.
Benihana was having financial difficulties. The directors got together with the CEO
and the general counsel and worked out a plan to issue $20M or preferred stock. A
company called BFC stepped up to buy the stock. The stock deal was partially
negotiated by a Benihana director named Abdo. Abdo just also happened to be the
principle owner of BFC. The directors approved the stock sale. Aoki filed a
derivative lawsuit against the directors.
● Issue: Aoki argued that the directors had breached their fiduciary duties by allowing
Abdo to negotiate the deal from both sides. That would be self dealing, which is a
breach of the duty of loyalty.
● Holding:
The Delaware Supreme Court looked to Delaware law (DGCL §144) which
provided safe harbor for interested transaction if "the material facts as to the
director's relationship or interest and as to the contract or transaction are disclosed
or are known to the board of directors, and the board in good faith authorizes the
contract or transaction by an affirmative vote of the majority of the disinterested
directors."
1. Basically, if the directors know about the conflict, and the majority of the ones uninterested
still vote to allow the transaction, then it is protected by the business judgment rule. If you’re an
interested director, must disclose to the other directors your position in order for a valid vote. The
uninterested directors must vote in good faith for a valid vote.
● Aoki argued that §144 was not applicable because the directors didn't know that
Abdo negotiated the deal, but the Court found that didn't really matter because the

43
directors already knew that Abdo, as the principle of BFC, would have to approve
whatever deal had been negotiated.
● So §144 still applies. The Court found that since the directors spent a lot of time on
the process of making their decision, and that transaction was a fair deal that was
approved by a majority of disinterested directors, it is covered by the business
judgment rule, and won't be overturned unless it can be shown that it amounts to
corporate waste.
● Corporate waste can be defined as "an exchange of corporate assets for
consideration so small as to lie beyond the range at which a reasonable person might
be willing to trade
○ Corporate Opportunity Doctrine
● Delaware Test (Guth v. Loft)(Factor test.. Not an element test)
○ If there is a business opportunity presented to a corporate officer or director
which the corporation is:
■ 1)financially able to undertake,
■ 2)in the line of the corporation’s business,
■ 3)one in which the corporation has an interest or a reasonable
expectancy, and
■ 4)by embracing the opportunity, a conflict of interest arises between
the officer/director and the corporation,
○ Then the law will not permit the director/officer to seize the opportunity for
himself
● For Guth Test, if there is a corporate opportunity, then decide what you do with the
opportunity and what are the consequences? If it is a corporate opportunity, you
need to offer it to the corporation. If it is not a corporate opportunity, then you don’t
have to offer it to the board.
● A director does not have to present the opportunity to the BOD to be protected by the
corporate opportunity doctrine. If a director believes that the corporation is not
entitled to the opportunity based on one of the factors, then the director may take the
opportunity for himself. (Broz v. Cellular Information Systems)
● Two Step Analysis for Corporation Opportunity Doctrine
○ Is there a corporate opportunity?
○ If it is a corporate opportunity…
■ a. Disclosure to the board and rejection by the board is the procedure
you follow
■ b. A formal presentation and disclosure to the Board is not required,
but does provide safe harbor.
● Broz v. Cellular Information Systems
○ Facts: Broz (P), the Pres and sole shareholder of RFB cellular utilized a
business opportunity for his wholly owned corporation instead of Cellular
Information Systems (D) for which he served as a member of the board of
directors.
○ Holding: Broz did not usurp a corporate opportunity by taking the license.
■ It was not clear the license was in CIS’s interest or expectancy
because CIS was divesting all of its cellular holdings and were not
looking to make new acquisitions.
44
■ Broz did not compete against CIS for the license, only PriCellular
■ Broz was not required to present the opportunity to the BOD
■ Broz had no duty to consider the contingent plans of PriCellular
● In re eBay, Inc. Shareholders Litigation
○ Facts: In 1995, Omidyar and Skoll founded the online auction community,
eBay, as a sole partnership. In 1998, eBay retained investment banks to
underwrite an initial public offering of public stock. Goldman Sachs was the
lead underwriter. The stock became immensely valuable. Goldman Sachs
rewarded favored clients, including some directors and officers of eBay, by
allocating to them thousands of IPO shares at the initial offering price,
resulting in huge instant profits. Certain other shareholders of eBay stock (Ps)
filed suit against those eBay directors and officers (Ds) for usurping a
corporate opportunity. Ds move to dismiss these claims for failure to state a
claim.
○ Holding: The directors usurped a corporate opportunity.
○ Analysis: Application of Guth test:
■ eBay was financially able to exploit the opportunity.
■ eBay was in the business of investing in securities – it had invested
hundreds of millions in securities over the years – and the corporation
was never given an opportunity to accept or decline the IPO
allocations.
■ This was not just an “advantageous investment opportunity” or “a
broker’s investment recommendation” that came to the directors.
■ The transaction created a conflict of interest with the directors and
their duties to the corporation (IPO allocation was intended to
persuade directors to choose to take eBay’s business to Goldman in
the future)
■ Agency theory of liability: Even if the
allocations did not constitute a usurpation of a
corporate opportunity, the directors breached
the duty of loyalty by failing to account for
profits obtained personally in connection with
transactions related to the company
● Sinclair v. Levien
○ Facts: Levien (P), a minority shareholder of Sinven, the subsidiary of
Sinclair (D), brought a derivative action against Sinclair (D) for causing
Sinven to pay excessive dividends, to breach Ks, and to forfeit corporate
opportunities all to the detriment of minority shareholders
○ Holding: Where a fiduciary duty governs a business transaction, the intrinsic
fairness test will be applied ONLY IF the transaction is accompanied by self-
dealing. SO, general rule for business transactions = business judgment rule
applies; IF duty of loyalty claimed, however, use intrinsic fairness test.
■ a. Exception to the rule that only directors hold a fiduciary duty. The
fiduciary duty of the shareholder, the parent company, arrives from

45
the domination and control of the subsidiary by the parent
corporation.
○ Test: The standard of intrinsic fairness involves both a high degree of fairness
and a shift in the burden of proof.
■ a. Under this standard the burden is on the parent company to prove,
subject to careful judicial scrutiny, that the transaction(s) with
subsidiary is objectively fair to minority shareholders.
● i. When the situation involves a parent and a subsidiary, with
the parent controlling the transaction and fixing the terms, the
test of intrinsic fairness, with its result shifting of the burden
of proof, is applied. Self-dealing occurs when the parent, by
virtue of its domination of the subsidiary, causes the
subsidiary to act in such a way that the parent receives
something from the subsidiary to the exclusion of, and
detriment to, the minority stockholders of the subsidiary.
■ b. Otherwise, if self-dealing is not involved/claimed, then BJR will
govern the business transaction, in which places the burden on
challenging shareholder to rebut the BJR.
● Ratification §144(a):
○ Interested director/officer transaction not void or voidalbe if (1) Material fact
discolsure and disinterested director approval;
○ or (2) material fact discolsure and shareholder approval;
○ or (3) contract is fair.
● Fliegler v. Lawrence
○ Facts: The shareholders of Agau (P) brought derivative suit against
Lawrence (D), president of Agau, alleging that he usurped a corporate
opportunity that belonged to Agau. The board rejected the corporate
opportunity and a majority of shareholders approved the rejection.
○ Holding: Ratification of an “interested transaction” by a majority of
independent, fully informed shareholders shifts the burden of proof to the
objecting shareholder to demonstrate that the terms of the transaction are
intrinsically unfair, i.e. so unequal as to amount to a gift or a waste of
corporate assets.
■ NOTE: Ordinarily, in transactions where there is a viable duty of
loyalty claim, director/officer has burden of proving that transaction
was intrinsically fair for it to be upheld over shareholder objections →
ratification switches this rule
○ To be an effective shareholder ratification, such that burden of proof shifts to
the challenging shareholder, approval MUST be granted by a Majority of
Independent, Fully informed Shareholders.
■ The Obligation of Good Faith
● Good faith:
○ Cede & Co. v. Technicolor, Inc., (Del. 1993)

46
■ Business judgment rule is “a powerful presumption” against judicial
interference with board decision making”
■ To rebut the rule, a shareholder plaintiff assumes the burden of
providing evidence that directors, in reaching their challenged
decision, breached any one of the triads of their fiduciary duty—
good faith, loyalty or due care.
■ If burden not met, business judgment rule attaches to protect
corporate officers and directors
■ If business judgment rule rebutted, burden shifts to to defendant
directors to prove to the trier of fact the “entire fairness” of the
transaction to the shareholder plaintiff.
○ 1. Courts often refer to the business judgment rule as “a presumption” that
the directors or officers of a corporation acted on an informed basis, in good
faith, and in the honest belief that the action taken was in the best interests of
the company
○ 2. DGCL § 141(e) provides: “A member of the board of directors, or a
member of any committee designated by the board of directors, shall … be
fully protected in relying in good faith upon [specified documents and
persons].... presented to the corporation by any of the corporation’s officers
or employees, or committees of the board of directors, or by any other person
as to matters the member reasonably believes are within such other person’s
professional or expert competence and who has been selected with reasonable
care by or on behalf of the corporation.”
○ 3. DGCL § 102(b)(7) provides that a corporation’s articles of incorporation
may (but need not) contain: “A provision eliminating or limiting the personal
liability of a director to the corporation or its stockholders for monetary
damages for breach of fiduciary duty as a director, provided that such
provision shall not eliminate or limit the liability of a director: … for acts or
omissions not in good faith or which involve intentional misconduct or a
knowing violation of law….
○ 4. DGCL § 145(a) and (b) only authorize indemnification of a director or
officer who “acted in good faith.”
○ Breach of duty of good faith if
■ 1) subjective bad faith (fiduciary conduct motivated by actual intent
to do harm to the corporation)
■ or 2) intentional dereliction of duties, conscious disregard for one’s
responsibilities
○ Gross negligence is not equal to bad faith
● In re Walt Disney Co. Derivative Litigation
○ Facts: Disney hired a new President and gave him a very high salary.
President wanted downside protection in the event the job didn’t work out.
Contract was negotiated to contain a clause that guaranteed Ovitz a severance
payment if he was fired for any reason other than gross negligence or
malfeasance. President didn’t work well with the company. The CEO first
tried to negotiate a trade and then explored terminating President. He was
terminated without cause after 14 months, but received a huge severance
payout. Disney shareholders brought derivative action against CEO and

47
Disney Board of directors claiming a breach of fiduciary duty and waste of
assets.
○ Holding:The defendants did not violate their duty of good faith because
approving and paying out the contract was a part of their contractual
obligations.
■ Analysis:
■ A failure to act in good faith requires conduct that is qualitatively different,
and more culpable than, the conduct giving qualitatively different from, and
more culpable than, the conduct giving rise to a violation of the fiduciary
duty of care (i.e. gross negligence).
■ Three categories of bad faith:
● 1)Subjective bad faith – fiduciary conduct motivated by an actual
intent to do harm.
● 2)Lack of due care – fiduciary action taken solely by reason of gross
negligence and without any malevolent intent.
○ Gross negligence by itself cannot constitute bad faith
● 3) Intentional dereliction of duty – a conscious disregard for one’s
responsibilities
■ A P who fails to rebut the business judgment rule presumptions is not entitled
to any remedy unless the transaction constitutes waste.
■ To recover on a claim of corporate waste, the Ps must shoulder the burden of
proving that the exchange was so one sided that no business person of
ordinary, sound judgment could conclude that the corporation has received
adequate consideration.
● A claim of corporate waste will arise only in the rare,
“unconscionable case where directors irrationally squander or give
away corporate assets.”
○ The payment of contractually obligated amount cannot
constitute waste, unless the contractual obligation is itself
wasteful
● Stone v. Ritter
○ Facts: AmSouth operated banks. AmSout was assessed millions in fines/civil
penalties based on failures of bank employees to file suspicious activity
reports required in regulations. Shareholders of AmSouth filed a derivative
complaint against 15 directors for their alleged lack of proper oversight.
○ Holding:
■ Generally where a claim of directorial liability for corporate loss is
predicated upon ignorance of liability creating activities within the
corporation, only a sustained or systematic failure of the board to
exercise oversight-such as an utter failure to attempt to assure a
reasonable information and reporting system exists—will establish the
lack of good faith that is a necessary condition to liability (from In re
Caremark Int’l Deriv. Litig.).
■ To find director oversight liability, directors must:
● (a) utterly fail to implement any reporting or information
system or controls; or

48
● (b) consciously fail to monitor or oversee its operations thus
disabling themselves from being informed of risks or
problems requiring their attention
● Securities Regulations
○ A. Regulatory Schemes
■ 1. Federal Securities regulations are designed to protect investors and the integrity of the
marketplace
● a. Done through
○ i. Securities Act of 1933- Primary Market
■ Regulates the capital markets that deals with the issuance of new
securities. The sale of a new stock or bond issue. Done through
underwriting/IPOs
○ ii. Securities Exchange Act of 1934- Secondary Market
■ Regulates secondary markets such as NYSE/Nadaq/Bond Markets
● b. Regulations consist of disclosure provisions, anti-fraud provisions, and regulations
of markets and market professionals.
○ i. If the key elements of these provisions are not fulfilled, actions can be seen
as fraud.
■ 2. Congress granted executive agencies (SEC) power through legislation. The rules of these
agencies carry the force of law.
● a. The SEC has been delegated the authority to make its own rules and to adjudicate
matters relating to securities. Strong independent authority.
● b. Has its own regulation and enforcement power—as congress would rather just
hand over power than pass a bunch of laws.
■ 3. State Blue Sky Laws
● a. Some states have regulatory laws that predate congressional/SEC action.
● b. Both federal and state law must be complied with, but there is often overlap.
○ B. What is a Security?
■ 1. Generally- The Security Act of 1933, Section 2(a)(1) (Robinson)
● a. Security” means, unless the context otherwise requires, any note, stock, treasury
stock, bond, debenture, evidence of indebtedness,...investment contract…or in
general any interest or instrument commonly known as a security.
● b. When something fulfills this definition, it is subject to the SEC’s disclosure and
antifraud provisions.
■ 2. Instruments fall under this provision
■ a. Investment Contracts (Howey) (SCOTUS 1946)
● i. Investment of Money
● ii. In a common enterprise
● iii. With the expectation of profits to come solely from the efforts of others
■ Stock (Landreth Timber) (1985)
● i. Pays Dividends
● ii. Negotiability (can be negotiated)
● iii. Can be Pledged or Hypothecated
● iv. Voting Rights
● v. Ability to Appreciate in Value
49
○ A. The labels the parties use have very little influence on whether
something is a stock.
○ B. So courts use the Economic Realities Test - court looks past the
labels to see what is going on
○ In determining whether it falls w/in the reach of securities law-what
matters more than the form of an investment scheme is the “economic
reality” that is represents
● Would the following classify as “stock”:
○ Shares of a closely held (private) corporation?
■ It would have been classified as a security - just like purchasing
shares
○ General Partnership interest?
■ Not a security - partners are active in management, so the third prong
of the stock test is never met (similar to a member managed LLC
interest)
○ Limited Partnership interest?
■ If the interest is as a limited partner, then it would be a security -
limited partners are not active in management of the partnership

○ Robinson v. Glenn (What is a security?)


■ Facts: Defendant contacted plaintiff about communications technology to raise capital.
Plaintiff invested in company, received shares, became treasurer, etc. However, technology
did not exist/hadn’t been implemented.
■ Plaintiff alleged securities fraud under 10(b)-5 (fraud provision), claiming that his interest
was a security (1933 act).
■ Holding: 1) Not an investment contract because he took an active part in management as a
member of the member managed (assumes active management of all members) LLC
security because he was not expecting the profits to come solely from others. 2) Although
the certificates were lacked the stock characteristics from the Landreth test and were not
called stock.
● C. Registrations and Exemptions from Registration
○ Securities Act of 1933, §5
■ a. Securities Act (1933) PROHIBITS the sale of securities unless the company issuing the
securities (issuer) has “registered” them with the SEC → must file registration statement
with SEC containing certain disclosures about the company’s finances/business, etc.
● MUST be approved by SEC before company can legally sell securities
■ b. Section 5 of the securities Act imposes three basic rules (1) a security may not be offered
for sale through the mails or by use of other means of interstate commerce unless a
registration statement has been filed with SEC; (2) securities may not be sold until the
registration statement has become effective; (3) the prospectus – a disclosure statement –
must be delivered to the purchaser before a sale
■ c. The SEC must approve the disclosures made in the prospectus before company can sell
the new securities.
■ d. The Securities Act includes two types of exemptions to the registration:
50
● (1) It exempt some securities entirely and
○ Example: US government securities
● (2) Exempt some transactions in securities not otherwise exempt.
○ a. In general, an exempt security need never be registered, either when
initially sold by the issuer or in any subsequent transaction. Exempt
transactions, in contrast, are onetime exemption.
○ Example: Private Placement
■ Remedy for selling unregistered securities is rescission
○ Securities Act of 1933, §4(2) – The provisions of Section 5 shall not apply to transactions by an
issuer not involving a public offering
○ No definition of public offering in the Securities Act of 1933
○ Ralston Purina test: Did the offerees need the protection of the registration provisions of the 1933
Act or were they able to fend for themselves?
■ Turns on access to information
■ Four factors to consider (Doran)
● Number of offerees and relationship to each other and issuer
○ Low number of offerees indicates private placement
○ Offerees that are sophisticated investors indicated private placement
○ Must be sufficient basis of accurate info upon which the sophisticated
investor may exercise his skills/knowledge for a private placement.
● Number of units offered
● Size of the offering
● Manner of the offering
○ Doran v. Petroleum Management
■ Facts: Petroleum Mgmt. Corp. (D) organized a limited partnership. (D) contacted Doran
(P) to see if he would participate and (P) agreed. Because Petroleum (D) violated some
ordinances and (P) not receive the profits he expected, Doran (P) sued Petroleum (D) to
rescind. Doran (P) claimed that Petroleum (D) violated federal securities laws (Section 5 of
1933 Act) by faililng to file a registration statement for securities old through interstate
commerce
■ Holding: Section 4(2) exempts private offerings from the registration requirement. The term
private offering is not defined in the Act, so the scope of the exemption must be determined
by reference to the legislative purpose of the Act. (See factors above).
● D. Fraud in the Registration Statement
○ There are two parts to a registration statement – the financial statement and narrative part, which
includes basic information of the corporation.
○ Section 11 of the 1933 Act:
■ Material misstatement or omission in a registration statement is actionable fraud
● Person acquiring such security has express private right of action
● Against any person who signed registration statement, any director, any expert
(accountant, engineer, appraiser, but not lawyer) who prepared or certified part of
registration statement, underwriters
● Defenses:
○ Loss causation: the lost that was experienced by the plaintiff was caused by
something other than the misstatement or omission

51
○ Due diligence: did your homework. Reasonable basis for your belief on what
you were signing was fair and full disclosure
● Remedy = damages
○ Materiality in a Registration Statement (Escott)
■ Definition: information that an average prudent investor ought reasonably to be informed
about before purchasing the registered security

○ Due Diligence Defense under Section 11 of the 1933 Act Defense

Expertised portion Nonexpertised portion

Expert Must conduct a “reasonable No liability


investigation” and believe
information is true

Nonexpert Must have no reason to Must conduct a “reasonable


believe information is false. investigation” and believe
information is true.

■ Section 11(c) “Reasonable Investigation”: standard of reasonableness required is that of a


prudent man in the management of his own property
■ Section 11 Serves DETERRENCE function
● Section 11 reaches material misstatements or omissions in registration statement
● Section 12 reaches material misstatements or omissions outside of a registration
statement
● Section 11 and 12 create express private rights of action
○ Compare implied private right of action (Section 10(b) of 1934 Act)
○ Escott v. BarChris
■ Facts: Misrepresentations in registration statement. Liability as follows:
● Directors: Does not turn on whether they understood statement, just that they did/did
not do a reasonable investigation. Could have partially relied on the expert portion
from the auditors, but the directors were still liable for the non-expertised portion.
● CFO/Treasurer: Dishonesty with people he relied upon not a valid defense.
● In house counsel: Should look into the validity of a prospectus/statement.
● Outside director: No investigation into prospectus not ok.
● Accountants: Reasonable investigation for individual in field. Individual auditor in
this case also shouldn’t have just relied on the info relayed to him; conflicting info
should have caused him to verify underlying info.
■ Holding:
● Were there material misstatements or omissions in the registration statement?
● Did everyone who singed the registration statement, every director,
accountant/engineer/other expert do a reasonable investigation?
○ D. Private causes of action.
■ Section 10(b) of the Securities Exchange Act of 1934: “It shall be unlawful for any
person..to use or employ, in connection with the purchase or sale of any security…any
manipulative or deceptive device or contrivance in contravention of [rules and regulations of
the Securities and Exchange Commission”
■ Rule 10b-5 of the Securities Exchange Act of 1934: “It shall be unlawful for any person…

52
● (a) to employ any device, scheme or artifice to defraud,
● (b) to make any untrue statement of a material fact or [material omission], or
● (c) to engage in any act, practice, or course of business…which operates as a fraud
or deceit upon any person”
○ Types of Cases: defective corporate disclosure, insider trading, fraud in dealings between broker-
dealer & their customers
■ Implied private right of action
● Well-established cause of action
■ Elements of Private Right of Action:
● material misrepresentation or omission, scienter (intent or knowledge of
wrongdoing), reliance, causation, damages
○ Fraud
■ Basic v. Levinson
■ Facts: Levinson (P) alleged that Basic’s (D) repeated denials regarding its intent to merge
with another company were material misrepresentations which Basic shareholders relied on
and were harmed by in later selling the stock at a discounted price.
■ Holding:
● 1) Whether a company statement is material, in the context of merger discussions,
requires a case-by-case analysis of the probability that the transaction will be
consummated and the significance of the transaction to the issuer of the securities.
● 2) An investor’s reliance on material, public misrepresentations may be presumed
under a fraud-on-the-market theory for purposes of a Rule 10b-5 action.
○ a. Reliance requirement → satisfied by fraud-on-the-market theory, i.e. an
investor who buys or sells stock at the price set by the market can be
presumed to have relied on any material misrepresentations affecting that
price
■ i. Fraud-on-the-market reliance may be overcome by D by showing
that the trading market was not efficient; the challenged
misrepresentation did not affect the stock’s price; the particular P
would have traded regardless of the misrepresentation
○ b. Materiality requirement →
■ Total mix test: Historical information (TSC Industries)
● “An omitted fact is material if there is a substantial likelihood
that a reasonable shareholder would consider it important in
deciding how to vote.”
● Substantial likelihood that it would have altered the ‘total mix’
of info made available.
■ Probability magnitude test: Speculative information
● Materiality will depend at any given time upon a balancing of
both the indicated probability that the event will occur and the
anticipated magnitude of the event in light of the totality of the
company activity.
○ Probability: look to indicia of interest in the
transaction at the highest corporate levels (examples:
board resolutions, instructions to investment bankers,
53
and actual negotiations between principals or their
intermediaires)
○ Magnitude: size of the two corporate entities and the
potential premiums over market value
■ Santa Fe Industries v. Green
● Facts: Defendant acquired a 60% share in Kirby in 1934. By 1974, Defendant
owned 95% and wanted to own the entire 100%. Defendant utilized Delaware’s
short-form merger statute that allowed a parent corporation owning at least 90% of
the stock to merge with the subsidiary and force the minority shareholders to sell
their shares. The minority shareholders must be notified within ten days, and
Defendant did so in this case. Defendant offered $150 per share after it was valued
by Morgan Stanley at $125. However, Kirby’s assets were valued to be $640 per
share. Delaware law allows a minority shareholder to petition the Delaware Court of
Chancery if they believe the payout is unfair. Instead, Plaintiffs brought an action
under federal law, claiming that the majority owed a fiduciary duty to the minority,
and that breach violated Rule 10b-5.
● Holding: 10b-5 creates cause of action only for fraud & deception, not for unfair
corporate transactions or breaches of fiduciary duties. So Ps’ action failed.
● Insider Trading
○ a. Definition of Insider Trading: Anyone who, trading for his own account in the securities of a
corporation, has access, directly or inherently, to information intended to be available only for a
corporate purpose and not for the personal benefit of anyone may not take advantage of such
information knowing it is unavailable to those with whom he is dealing.
■ SEC v. Texas Gulf Sulphur
● Facts: D mining company learned of potentially profitable property. Corporate
officers purchased stock prior to public disclosure. D issued press release
downplaying find. Plaintiffs alleged insider trading and fraud in corporate disclosure.
● Holding: The information about the mineral find was not immaterial as a matter of
law.The issuance of the press release maybe violated rule 10b-5.
○ a.An insider is not of course always foreclosed from investing in his own
company merely because he may be more familiar with company operations
than are outside investors.
○ b. Materiality of inside information in insider trader: See fraud standard
of materiality in Basic (total mix test for historical information and
probability-magnitude test for speculative)
○ c. Disclose or abstain rule: Anyone in possession of material inside
information must either disclose it to the investing public, or, if he is disabled
from disclosing it in order to protect a corporate confidence, or he chooses
not to do so, must abstain from trading in or recommending the securities
concerned while such inside information remains undisclosed.
○ Tippee Liability
■ Dirks v. Securities and Exchange Commission
● Facts: Dirks (D) was an officer of a broker-dealer firm that specialized in providing
investment analysis. He received information from an officer of EF that EF’s assets
54
were grossly overstated. Neither D nor his form owned or traded any EF stock.
However, D discussed his findings with a number of clients and investors, some of
whom subsequently sold their EF stock. The price of EF stock dramatically declined.
● Holding: Dirks did not violate Rule 10b-5 because Secrist (the tipper) did not obtain
an advantage for himself in disclosing the info, and therefore there was no breach of
fiduciary duty by Secrits.
● Tippee Liability
○ Test for whether a party can be held liable for acting on a tip:
■ 1) Did the Tipper breach a fiduciary duty?
● Existence of breach turns on receipt of direct personal benefit
by tipper
● Mere possession of material inside info does not give rise to a
duty to disclose
● Temporary insiders may include: accountants, lawyers,
underwriters, or consultants.
○ These people may become fiduciaries of the
shareholders
■ 2) Did the Tippee know or should the tippee have known of the
Tipper’s breach
● Hypos - Would Dirks have been liable under these alternative scenarios:
○ If Dirks and Secrist had routinely exchanged stock tips?
■ Dirks would have liability here because Secrist would be receiving a
direct personal benefit in the form of stock tips from Dirks
○ If Secrist had disclosed the EF fraud in part because he had been fired over an
unrelated matter?
■ Dirks may be liable here because although he doesn’t financially
benefit from the disclosure, he benefits psychologically from the
demise of his former employer
○ If Dirks over heard Secrist describing the fraud to another person in an
elevator?
■ Dirks would not be liable here because Secrist is just being careless
and won’t receive a benefit from the disclosure
○ What if Secrist disclosed negative inside info (not involving fraud) because
Dirks bribed him. Dirks then advised his clients to sell their EF stock. Dirks
would have violated Rule 10b-5. Would his clients also have violated the
rule?
■ Yes
○ Outsider trading prohibition
■ MISAPPROPRIATION THEORY
■ Outsider violates 10b/10b-5 when he trades on material non-public information in
breach of a duty owed to the source of such information
○ Disclosure to source of information absolves breach
○ 10(b) requires “deceptive” conduct “in connection with” securities
transactions. If outsider feigns fidelity to the source of the confidential
55
information, while secretly converting the info for personal gain (purchasing
and selling), constitutes deception (if outsider had disclosed to the source that
he planned to trade on the info, there would be no violation of 10(b) because
there was no deception).
○ Purpose: protect integrity of markets against abuses by outsiders who have
access to confidential info that will affect a company’s stock price but who
owe no fiduciary duty to corporation’s shareholders
■ Section 14(e) of 1934 Act & SEC Rule 14e-3(a): Insider Trading Prohibition in Tender
Offers
● Section 14(e): In connection with a tender offer, it shall be unlawful to make
material misstatements or omission or to engage in fraud, deception or manipulation
● Rule 14e-3(a): If a tender offer has been commenced, it is unlawful to purchase or
sell securities on the basis of material inside information if trader knows info
obtained from offeror, issuer or any officer, director, partner, or employee to either
offeror or issuer
■ US v. O’Hagan
● Facts: Respondent was a partner in a law firm, Dorsey & Whitney, which was
representing a company that was potentially tendering an offer for common stock of
the Pillsbury Company. Respondent was not personally involved in the
representation, but he was aware of the transaction enough to know that if he
purchased Pillsbury securities now that they would increase in value once the offer
went through. Respondent was going to use the profits from this transaction to
replace money that he embezzled from the firm and its clients. After the offer went
through, he made a $4.3 million profit. The SEC investigated Respondent’s
transactions and claimed he violated Section:10(b) and Section:14(e) for
misappropriating confidential information.
● Holding: Respondent violated Section 10(b) and Rule 10b-5 when he
misappropriated nonpublic information to personally benefit through the trading of
securities.
○ The “in connection with” requirement is satisfied. D’s fraud was
consummated, not when he gained the confidential info, but when he used the
info to purchase and then sell securities. The person or entity defrauded does
not have to be the other party to the transaction
○ Court said that had O’Hagan made disclosure to the company his firm
represented → No liability.
○ Short-Swing Profits
■ §16(b) of the 1934 Act: Short-Swing Profits (Disgorgement Rule)
● A public corporation may recover
○ Realization of any profits
○ By an officer, director, or 10% shareholder
○ From matching purchases or sales during any six month period
○ Of equity securities of the public company
● Purpose of the rule was to prevent insider trading. However, rule is highly arbitrary,
leading to many innocent parties violating the rule and guilty parties to escape it
56
■ Reliance Electric v. Emerson Electric
● Facts: D acquired 13.2% out the outstanding common stock of Dodge in an
unsuccessful attempt to take over Doge. The shareholders of Doge approved a
merger with P. D realized that any further attempts to take over Dodge were certain
to fail and decided to dispose of its shares. On the advice of counsel, D decided to
sell enough stock to bring its holdings under 10% and thereby immunize the
remainder of their shares from liability under 16(b).
● Holding: D was allowed to sell off the stock in two separate transactions, within 6
months, in an attempt to immunize the second transaction from liability under
section 16(b). Liability cannot be imposed on D because its purpose was to avoid
liability.
● Indemnification and Insurance
○ GENERALLY, corporations may have a duty under statutory law to indemnify officers and/or
directors for any damages they might incur in connection with their corporation activities.
○ Indemnification
■ This is the corporation’s promise to reimburse the director for litigation expenses and
personal liability if the director is sued because she is/was a director. Director’s
indemnification rights continue even after left the corp.
■ Because of indemnification’s potential to frustrate other goals and polices, a director’s right
to indemnification and the power of the corp to indemnify depend on whether
● The director was successful in defending the action
● The director, although, unsuccessful in the defense, was justified in his actions
■ If the director is sued b/c of corporate position and she defends successfully, the corp is
obligated under all state statutes to indemnify the director for litigation expenses, including
attys fees
■ When is a defense successful?
● Corporate statutes uniformly require indemnification when the D is successful “on
the merits,” such as when the suit is dismissed for lack of evidence or on a finding of
non-liability after trial
● Under most statutes, success can also be on procedural grounds—success
“otherwise”—such as when the suit is dismissed b/c the Pl lacks standing or the SOL
has run
○ §145 DGCL. Indemnification of officers, directors, employees and agents; insurance
■ (a) Indemnification permitted in action brought by third party. This covers reasonable
litigation expenses and personal liability arising from court judgment, an out of court
settlement, or imposition of penalties or fines. This does require
● (1) acted in good faith – director did not know her conduct was illegal and did not act
for improper personal gain, and
● (2) reasonably believed the actions were in the corporation’s best interests
■ (b) Derivative suits – reasonable litigation expenses and good faith and reasonable belief
requirements
■ (c) Indemnification required if successful on the merits

57
■ (e) Allow corporation to advance litigation expenses during the proceeding… upon written
receipt of an undertaking to repay such amounts if ultimately determined that such person
not entitled to indemnification
■ (f) Additional rights to indemnification and advances may be granted by contract
■ (g) Corp. may purchase insurance for liabilities of covered, regardless of whether the corp.
would be able to indemnify the covered person.
○ Waltuch v. Conticommodity Services
■ Facts: Waltuch (P) sought indemnification of his legal expenses from his former employer,
Conticommodity (D) after lawsuits against him were dismissed. P claimed that D’s articles
of incorporation require D to indemnify him for his expenses in both actions because the
articles did away with the good faith requirement of section 145. D argued that P’s claim is
barred because section 145 of DGCL permits indemnification only if the corporate officer
acted in good faith.
■ Holding:
● Courts have held that success or vindication under §145(c) does not mean moral
exoneration. Escape from an adverse judgment, for whatever reason, is
determinative
● Contracted indemnification rights cannot be inconsistent with section 145’s
substantive provisions, otherwise, 145(g) would be rendered meaningless (which
allows a corp. to circumvent the good faith requirement by purchasing insurance to
cover directors/officers in situations which the corp. could not indemnify them
● Proxies and Proxy Fights
○ What IS a proxy → A document representing the right to act on the shareholder’s behalf, held by
a “proxyholder,” who is the shareholder’s agent and attends meetings/votes on shareholder’s behalf
■ §216 DGCL: need a quorum of shareholders here in order to conduct business
● Shareholders are typically very passive and don’t attend annual meetings and if the
corporation’s management cannot get a quorum, they cannot conduct business or
elect directors. This is the reason for proxy solicitations. People will get a copy of a
proxy card and proxy solicitation materials asking them to appoint someone
recommended by the corporation to vote on their behalf at the meeting
○ Purpose – to enable the corporation to have enough shareholders present, either physically or
through proxy to satisfy quorum
■ Often used in proxy to gain control of corporation
■ Proxy fights RESULT WHEN an insurgent group tries to oust incumbent managers by
soliciting proxy cards and electing its own representatives to the board → subject to 1934
Securities Exchange Act
○ Regulation: State Law
■ Statutory
● DGCL §211(b): actual requirement that shareholders meet
○ Requires annual meetings of shareholder for election of directors
○ Uncontested case: expenses always recoverable in order to get people into the
meeting
○ Contested cases: talk about who is going to be reimbursed
● DGCL §212(b)
58
○ Permits voting by proxy
■ Case law on strategic use of proxies and cost reimbursement in proxy contests
● Levin v. MGM (Strategic Use of Proxies)
○ Facts: Levin (P) and stockholders owning 11% of common stock of MGM
(D) brought suit against MGM and five of its board directors for using
corporate funds to pay for special attorneys, a public relations firm, and a
proxy solicitation organization in a proxy solicitation contest between the
Levin group and the O’Brien group. What this use unfair or illegal?
○ Holding: No. Incumbent directors may use corporate funds and resources in
a proxy solicitation contest if the sums are not excessive and the shareholders
are fully informed.
■ Limits: must be genuine issue of corporate policy, reasonable amount,
and cannot use unfair or illegal means
■ Must determine whether it a policy issue at stake or a personality
issue: Cannot get reimbursed for these expenses if it is a personality
issue. There must be a genuine fight over corporation policy, not
simply for keeping incumbents on the board. This enables and ensures
shareholders are fully informed of the competing corporation policies
and can thus make a decision
● Rosenfeld. Fairchild Engine & Airplane Corp. (Reimbursement of Cost)
○ Facts: Rosenfeld (P), a shareholder of Fairchild Engine, filed this derivative
suit to compel the return of funds paid out of the corporate treasury to
reimburse the expenses of both sides following a proxy context. The old
board of directors spent $106,000 in defense of their position while they were
still in office. The new board then paid the old board $28,000 following the
change in management to compensate the old directors for their remaining
expenses which the new board found to be fair and reasonable. Another
$127,000, which represented reimbursement of expenses to members of the
new board, was ratified by a 16 to 1 vote of the shareholders.
○ Holding: Directors may seek reimbursement from the corporation for
reasonable and proper expenses they incurred in a proxy contest.
■ Test for Incumbents (old board) - there was a genuine issue of policy
which the shareholders should be informed of, no fraud, and amount
was fair/reasonable. (reimbursement “test” is easier)
■ Test for Insurgents (new board) – shareholder ratification, the
shareholders approved. (higher burden)
○ Regulation: Federal Law
■ §14(a) of 1934 Act
● Requires proxy solicitations for reporting companies to comply with SEC rules
● SEC rules require:
○ Disclosure to accompany proxy solicitation
○ Specific form of proxy card
○ Require prior filing and review of proxy statement and proxy card
○ Prohibit false and misleading proxy solicitations
59
■ SEC Rule 14a-8: Shareholder Proposals (discusses fraud - unlawful to do a proxy
solicitation and engage in fraud)
● Definition - recommendation or requirement that company and/or board take action
which you intend to present at meeting of shareholders
● Company must include info in proxy statement and identify it in form of proxy
subject to procedural limitations and the right to exclude
○ Procedural limitations
■ Must own $2000 or 1% of stock
■ May submit on proposal not to exceed 500 words
○ If the proposal is proper the management must include it in company’s proxy
mailing to shareholders unless it can meet burden of showing proposal may
be excluded for one of the reasons set forth in question 9
○ If management decides not to include the proposal, then it must notify
shareholder and allow opportunity to correct, must file reasons with SEC and
this is challengeable
■ SEC Rule 14a-7: Common Carrier Obligation
● Management must mail shareholder materials or provide shareholder with a list of
names and addresses of other shareholders.
○ Shareholder Inspection Rights
■ Basically, involves the shareholder who wants to use proxy to change board who needs
support of major shareholders → has to get “shareholder list” to find those major
shareholders.
■ NYBCL §1315
● Access must be permitted to qualified shareholders on written demand, subject to
denial if the petitioner refused to furnish an affidavit that the inspection is not desired
for a purpose other than the business of the corporation and that the petitioner has not
been involved in the sale of stock lists within the last 5 years.
■ Crane v. Anaconda Co.
● Facts: Crane (P) announced a proposed offer to exchange subordinated debentures
for shares of common stock of Anaconda (D). D opposed the tender offer and sent
four letters to its shareholders asserting that he offer was not in the best interests of
the company. P requested a list of D’s shareholders claiming that D had a fiduciary
duty to present shareholders with all pertinent information regarding the pending
tender offer. D refused to provide the list because it felt P’s reasons for the request
were not purposes relating to the business of D within the meaning of NYBCL
§1315 . P filed this petition to compel D to produce its shareholder list
● Holding: A shareholder may access a shareholder list for the purpose of informing
other shareholders of its exchange offer and soliciting of stock
○ A qualified shareholder is allowed, when in good faith, to inspect a
corporation’s stock register in order to notify shareholders of exchange and
solicitation offers for stock
○ A shareholder desiring to discuss relevant aspects of a tender offer should be
granted access to the shareholder list unless it is sought for a purpose inimical

60
to the corporation or its stockholders —and the manner of communication
selected should be within the judgment of the shareholder
■ DGCL Section 220
● A stockholder (or through an atty/agent) can examine the following upon written
demand under oath :
○ The corporation's stock ledger, a list of its stockholders, and its other books
and records; and
○ A subsidiary's books and records
● A proper purpose shall mean a purpose reasonably related to such person's interest as
a stockholder.
■ Compare NYBCL §1315 and DGCL §220
● DG: Must always show proper purpose to inspect ledger, list of its stockholders,
books/records. Can be any stockholder or their agent.
● NY: Unqualified right to inspect annual balance sheets and profit and loss
statements. However, must show good faith and proper purpose for all other records
(ie- list of stockholders!). (Statute may indicate that a former shareholder has this
right, “Any resident of this state who shall have been a shareholder of record.” Can
also use agents)
■ Pillsbury v. Honeywell
● Facts: A shareholder (P) bought 100 shares in Honeywell (D) for the sole purpose
of giving himself a voice in D’s affairs so that he could persuade D to cease
producing munitions used in the Vietnam War. P demanded access to D’s
shareholder list and all corporate records dealing with weapons and munitions
manufacture, for the purpose of communicating with other shareholders to elect a
new board of directors who would represent his viewpoint. D refused to provide the
documents, and P filed suit to compel production
● Holding: D was not required to produce the list
● Rule: A shareholder can only demand corporate investor identification information
when the purpose is related to investment concerns traditionally associated with
shareholder concerns

61

You might also like